You are on page 1of 75

KAPLAN LSAT PREP

LSAT
RELEASED TEST III
EXPLAINED
A Guide to the December, 1991 LSAT
KAPLAN
The answer to t he test quest i on.
1995 Stanley H. Kaplan Educational Center Ltd
All rights reserved. No part of this book may be reproduced in any form, by
photostat, microfilm, xerography or any other means, or incorporated into any
information retrieval system, electronic or mechanical, without the written permission
of Stanley H. Kaplan Educational Center Ltd.
K A P L A N 1
SECTION I:
LOGIC GAMES
LSAT PREP ________________________________________________________________ LSAT Test III Explained: Section I
2 K A P L A N
GAME 1Three Couples At Dinner
(Q. 1-7)
The Action: We can tell from the first three sentences that the first game is a matching
game; we're given 6 people (3 couplesJohn and Kate, Lewis and Marie, and Nat and
Olive), and are asked to match each to one particular entree (pork chops, roast beef,
swordfish, tilefish, or veal cutlet). Notice that with 5 entrees and 6 people, there has to be
some duplication of entrees. The Key Issues are basic:
1) Who orders what entree?
2) Who can, must, or cannot order the same entree as whom?
The Initial Setup: To sketch the information, simply write J, K, L, M, N, and O across the
top of the page with double lines between K and L, and M and N, in order to visually
break the 6 people into their respective couples. If you wish, you can also distinguish
between men and women, by writing tone group in capital letters, and the other in
lowercase. Add in the list of the entrees off to the side, and you should have something like
this to begin with:
PRSTV
JK LM NO
The Rules:
Start with the two most concrete rules, Rules 3 and 5.
3) An S under Marie will remind us that she opted for the swordfish.
5) R under Olive means that its roast beef for her.
1) This rule adds a nice touchit allows the man and woman in each couple to share. One
easy way to remember this is to write a between the members of each couple.
2) You could write men never order the same, but youre always better off being more
specific: JLN.
LSAT PREP ________________________________________________________________ LSAT Test III Explained: Section I
K A P L A N 3
4) No fish for John or Nat, which means that swordfish and tilefish are out for them. But
there are only five entrees to begin with, which means that these two guys are restricted
to either pork chops, roast beef, or the veal. Indicate that somewhere in your sketch.
Key Deductions: and we have NO R B under Nat to indicate that he wants something
different than his date, Olive. Our work with combining Rules 2 and 4 will prove quite
useful. We should also include Rule 1 in our considerations.
Since Olive, Nats better half, orders roast beef (Rule 5), Nat cant, and Nats choices were
limited to begin with (Rule 4). Therefore, Nat must order either pork chops or veal cutlet.
We can also deduce that Lewis wont order swordfish since he wont duplicate Maries
choice. Notice that while we have definite choices for two out of the three women, we also
have lots of information on the men, especially Nat. Once you get the setup to this point,
you can bet that more than a couple of answers will spring from John and Nat running out
of entrees to order thanks to Rules 1 and 2.
The Final Visualization: Heres what we have before moving on to the questions:
PRSTV
J K L M N O
MEN NOT SAME
J L N
S R
P/R/V No S P or V
The Big Picture:
If rules are given to you in the negative, turn them around to the positive. The fact
that John and Nat dont order swordfish or tilefish is not nearly as useful as
deducing that they MUST order one of the remaining entrees; pork chops, roast beef,
or veal cutlet.
Always think actively. Flagging potential major rules and thinking through
possibilities are just two techniques that can really save time and aid in conquering
the lions share of the questions.
Work with the most concrete rules first; anything that can go into the sketch
immediately will help to lay the groundwork for the rest of the Final Visualization.
But once theyre incorporated (Rules 3, 4, and 5 are now built into the visualization
above), turn your attention to the rules that will most likely govern the rest of the
game, which in this case are Rules 1 and 2.
LSAT PREP ________________________________________________________________ LSAT Test III Explained: Section I
4 K A P L A N
The Questions:
1. (D)
We deduced that Lewis will not order swordfish since Marie, his date, orders it. However,
theres nothing that prevents him from ordering any of the other entrees, so the other four
must be included in the list.
Dont be surprised if a relatively simple question starts off a game. The testmakers
are more than likely rewarding you for wading through the setup. Its also a good
test to see if you understand what does and DOESNT restrict an entity.
2. (B)
Not much to do but check out each choice. (A) is knocked out by a combination of Rules 3
and 4. (C) violates Rule 2. Rules 1 and 5, taken together, signify that (D) cant be true, and
Rule 1 by itself makes (E) impossible. That leaves (B); Kate would have to order either pork
chops or veal cutlet, because that's all Nat can eat, but thats eminently feasible.
Scanning the choices will often make the blatantly wrong choices stand out (for
example, seeing two men with the same entree or the members of a couple with the
same entree). Once these are eliminated, its simpler to go back and test those that are
left.
3. (A)
Heres our Big Deduction: Nat has to order either pork chops or veal cutlet, and choice (A)
therefore must be a true statement. (B) and (C) are impossible, while (D) could be true, but
need not be (Lewis could order tilefish) and (E) merely could be true as well, but would be
false if Kate ordered fish.
A non-if must be true question (one with no new information) is a big red flag that
there was a Big Deduction possible. When you dont initially see the deductions
possible in a particular game, very often the questions will force you into making
them. Its then your job to carry them with you for the rest of the game.
4. (E)
John orders the veal, so Nat and Lewis cannot (Rule 2). This makes pork chops Nats only
choice. Just for the record, Kate ordering pork chops and Lewis ordering roast beef kills
choices (A) through (D).
When youve done good deductive work, trust it. Dont figure out the answer and
then waste time trying out each answer choice. Scan the choices for your answer, and
if its there, mark it and move on.
LSAT PREP ________________________________________________________________ LSAT Test III Explained: Section I
K A P L A N 5
5. (C)
Nat doesnt order pork chops (no one does), so he orders veal cutlet. Oops, not one of our
choices, so we must continue. John now must order the roast beef (now his only choice).
Still not an answer choice (though it does rule out choice (A)). Lewis choices are now
limited to tilefish, whatever that may be. Thats a choice(C). (B) and (E) could be true, (D)
never.
Some questions in every game will require more than just the initial deductive step.
Dont worry. Keep plugging and youll find the answer.
6. (A)
Rule 2 forces Nat to order the veal cutlet, and the same rule mandates that John settle for
the roast beef. Johns complete and accurate list is simply roast beef, choice (A).
Its very common for several questions to test the same concepts. Thats why its vital
that you use your previous work. Questions do not exist in a vacuum, they all exist in
the context of the entire game. Thats why a question this far along in the game can
often be answered very quicklywere simply going over old ground.
7. (D)
Deal with rule changes first. Since Marie has swordfish, now so does Lewis. Olive has roast
beef; now, so does Nat. Rule 2 is still in effect, so John is left with a choice of pork chops
and veal cutlet, which means that Kate is limited to the same. They both can opt for the
pork chops, so (D) can be true.
(A), (B), (C), and (E) are impossible.
Dont just stare at a lengthy question; try it out. Use your pencil and work with the
answer choices. Even if you cant get an answer quickly (like in some of the early
questions on this game) doesnt mean that the point is lost. You can always eliminate
the wrong choices.
Sometimes its best to skip questions with rule changes, especially ones as lengthy
as this one. However, you should always take a quick look at the question before
delaying it, because sometimes, as in this case, the rule change actually makes the
question easier than most by narrowing the possibilities. Here, the rule change
allows us to quickly match up two more people to the food they order.
LSAT PREP ________________________________________________________________ LSAT Test III Explained: Section I
6 K A P L A N
GAME 2Seven Families in Houses
(Q. 8-13)
The Action: A fairly straightforward sequencing game, with a very minor wrinkle: the use
of the terms west and east to refer to the different sides of the sequence. Were asked to
order seven families (K, L, M, N, O, P, and R) in seven houses lined up west (left) to east
(right) on one side of a street. The Key Issues are:
1) Which families can, must, or cannot occupy which house?
2) Which family can, must, or cannot occupy a house adjacent to what other family?
The Initial Setup: Keep this setup simple; seven dashes or the numbers 1 to 7 from left to
right on your page will suffice. You may wish to jot down a W to the left and an E to
the right of the sketch, just for good measure, to remind you which side is which. List the
roster of families off to the side, and youre ready to fill the Rules into this basic setup:
1 2 3 4 5 6 7
KLMNOPR
W E
The Rules:
2) Once again, the most concrete rule first: The Kahns live in the fourth house from the
west. Dont let the awkward wording throw you. This simply means that the Kahns live
in house 4, so put that directly into the sketch.
1) No R over houses 1 and 7 should help us keep this in mind.
3) Since the Muirs live next to the Kahns, who live in house 4, the only possible houses for
the Muirs are 3 and 5. Place an M with arrows pointing to houses 3 and 5 into your
sketch.
4) Since the Kahns live in house 4, this rule tells us that the Piatts can only live in houses 5,
6, or 7. But the Piatts cant live in 7, because the Lowes must live to the east (right) of
them. So the P . . . L chunk will have to fall somewhere within houses 5, 6, and 7. Jot
that down, and were ready to move on.
LSAT PREP ________________________________________________________________ LSAT Test III Explained: Section I
K A P L A N 7
Key Deductions: Theres no major deduction here, but there are only a limited number of
ways the sequence could play out. Its therefore worth quickly exploring the possibilities:
M lives in either 3 or 5. Let's say M occupies the 5th house. Since the P. . . L pair must be
east of K, we get this:
1 2 3 4 5 6 7
O/N K M P L
R, O, and N are left to fill in the 1,2 and 3 houses. R, you remember, can't be on either end,
so that family would be in either 2 or 3 in this situation, leaving O and N for the other
spots, in either order.
The second scenario is a little more flexible, and arises if we instead place M 3rd. P and L
still need to be to the right of K, and must still maintain that K..P..L ordering. The other 3
N, O, and Rwould fill the remaining spots, so long as R isn't first or last. This ordering
isn't as determined as the other, and naturally doesn't help us as much, but it's still good to
consider the possibilities in advance, which we've done.
The Final Visualization: Heres what were armed with to tackle the questions:
1 2 3 4 5 6 7
KLMNOPR
W E
M
K
PL
no R no R
The Big Picture:
When one entity is restricted to two options, its usually worth it to try out the
possibilities. A rule of thumb for dealing with such options: If you can use the rules
to narrow down the concrete possibilities to two, it may be wise to get those
possibilities solidly down on paper. And when youve taken the time to do this,
remember to refer back to this work in the process of handling the questions.
Always strive to build a rule directly into your master sketch. Only when that is not
possible, should you rewrite the rule in your own words. When rewriting a rule is
not feasible, you should underline or circle it for future reference.
In such a straightforward game, the testmakers probably felt that they had the right
to include a slight variation on the sequencethis west to east business. But aside
from the language, this really doesnt change anything. Dont let a little wrinkle ruin
an otherwise simple game for you: Take command.
LSAT PREP ________________________________________________________________ LSAT Test III Explained: Section I
8 K A P L A N
The Questions:
8. (C)
The Lowes live in house 7 in our first possibility above, but thats not a choice. (A), (B), (D),
and (E) directly violate Rules 2, 3, 4, and 1, respectively, which leaves only (C). A glance at
our sketch would quickly confirm the same result.
This is simply another form of acceptability question, and given all we know, it
shouldnt have taken more than a few seconds to figure out. Recognize the various
types of non-if questions that are simply testing your understanding of the rules.
Keep track of free agents in the various games. In this one, only N and O arent
mentioned in the rules, so theyll be the ones with the most flexibility. For this
reason, such free agents are usually involved in could be true questions,
whereas the more restricted entities play a larger role in must be true questions.
9. (A)
This ones also pretty much a gimme. We can easily infer from Rule 4 that the Piatts live
between the Kahns and the Lowes. Therefore, theres no way that the Kahns can live next to
the Piatts. The rest of the families in (B) through (E) can live next to the Kahns.
Dont waste time trying out the choices after youve already found the answer. Trust
in your work, mark your answer, and move on.
10. (C)
The info in the stem puts M in house 3, which conjures up the second scenario we
discussed earlier. This was the less helpful scenario, so lets check out the choices.
(A) R can live next to both K and P: K in 4, R in 5, P in 6, and L in 7, with N and O floating
between 1 and 2.
(B) is possible too: M,K,P,R,L in 3 through 7, again with N and O floating at the beginning.
(C) M is in house 3, K is in 4, and P is either in 5 or 6. Its therefore not possible for the
Rutans to live between the Muirs and the Piatts, making (C) our answer.
(D) and (E) Either O or N can live in house 1, and with R in 2 and M in 3, so both of these
are possible.
It may seem time consuming to try out each choice, but doing so is an excellent way
to get familiar with the situations that will be played out in questions to come. If you
use your pencil to try out the various choices, it will often benefit you to come back
to your scratchwork in later questions.
LSAT PREP ________________________________________________________________ LSAT Test III Explained: Section I
K A P L A N 9
11. (A)
N in 3 points us to our first scenario:
1 2 3 4 5 6 7
N K M P L
Were left with R and O for houses 1 and 2, and Rule 1 forces R into house 2 and O into
house 1. The entire setup of families to houses is complete: O, R, N, K, M, P, L. The Owens
clearly do not live next to the Newmans. Choice (A) it is.
The testmakers tend to zero in on a few concepts that they consider important and
proceed to base the majority of the questions of these few concepts. All the better for
you, as long as you take the time to recognize them early on.
A question like this illustrates the value of playing out the various scenarios up
front; when everything quickly falls in to place, you buy extra time for the more
difficult questions.
12. (A)
Here we can deduce that the second scenario above is operative: The only way for the
Owens to live to the east, or right, of the Muirs, is for M to live in 3, with O, P and L fitting
somewhere in 5, 6, and 7. The two remaining entities, N and R, will therefore take houses 1
and 2, respectively (R cant take 1 from Rule 1). The choice that corresponds to this is (A): K
must be east of M. (B) is false, and (C) through (E) are possible only.
When given new pieces of information, immediately work them. Use your pencil, get
the new information down, and see where it leads.
It pays to focus on your bread and butter, which in this game is the placement of M.
This determines which scenario were dealing with, and leads to the correct answer.
Your job in every game is to find the critical entity or entities, and concentrate on
how theyre affected by each new piece of information.
13. (D)
The testmakers liked question 12 so much, here it is again (nearly). So we should consult
the same sketch as the one for number 12: M is in 3, K is in 4, O, P, and L are relatively free
to float between spaces 5, 6, and 7 (as long as P is before L), N is in house 1, and R is in
house 2. Scan down the list for adjacent families; N and R must live next to one another,
specifically in houses 1 and 2, respectively. (A), (B), and (E) contain pairs that are possible
neighbors, but need not be, whereas the pair in (C) are definitely separated by the Rutans.
Let previous work come to your aid whenever possible; be on the lookout for
question stems that virtually ask the same thing. Any time- saving technique will
help you rack up more points.
LSAT PREP ________________________________________________________________ LSAT Test III Explained: Section I
10 K A P L A N
GAME 3Car Display
(Q. 14-19)
The Action: This fairly complex game involves matching cars with 3 different
characteristics (family or sports car, new or used, and production models or research
models), and then with a fourth characteristic, the floor theyre displayed on. Important to
note that each floor contains all or one of each of the three different characteristics, i.e. there
will not be both new and used cars on a single floor. Theres only one major Key Issue that
every question will involve:
1) What characteristics are exhibited by the cars on each floor?
The Initial Setup: Since we're dealing with 3 floors each containing 3 characteristics, one
possible way to visualize this is to draw a 3 by 3 table. We can label the vertical side of the
box 3, 2, and 1, from top to bottom, and each box across horizontally would represent one
of the three characteristics: style, age and type. This way we can picture each floor and it's
relevant traits:
3
2
1
F/S N/U P/R
The Rules:
Where to start? As usual, with the concrete rules.
4) The cars on floor 1 are new. An N in the appropriate grid box will remind us that ALL
of the cars on floor 1 are new.
5) The cars on floor 3 are used. A U in the appropriate floor 3 grid box means that all the
cars exhibited there are used.
1) This rule is fairly lengthy, so take your time and be sure that you fully understand it. IF
(thats a big if) there are both family and sports cars, then all of the family cars will be on
lower floors than any of the sports cars. So if there are both kinds, floor 1 must be family
cars and floor 3 must be sports cars. Understanding this rule is far more important than
exactly how you choose to represent it on the page.
LSAT PREP ________________________________________________________________ LSAT Test III Explained: Section I
K A P L A N 11
2) There are NO cars that are both used and research models. What does this tell us? If a
car is used, then it must also be a production model. If a car is a research model, then it
must be new. If a car is new....nothing. Be careful, a new car can either be a research
model or a production model. If U, then P and if R, then N should help us
remember this.
3) None of the cars in the exhibition are both research models and sports cars. Again, be
careful when considering what this means. If a car is a research model, then it must be a
family car. If a car is a sports car, then it must be a production model. If a car is a family
car or a production model, then we know nothing (or at least nothing new). If R, then F
and if S, then P will aid in sorting it out.
Key Deductions: The results of our work with Rules 2 and 3 certainly qualify as Key
Deductions. However, there's even more to piece together. Rule 2 yielded if Research,
then New, and Rule 3 yielded if Research, then Family. To make it easier we can
combine those to get if Research, then New and Family. Also, dont neglect your grid.
Rule 5 resulted in a big U under floor 3, and thanks to Rule 2, we know that if Used,
then Production. Therefore, the floor 3 cars must be production models. Two of the
three characteristics are taken care of for that floor.
The Final Visualization: Heres what we have just before facing off with the questions:
3
2
1
F/S N/U
U
N
P/R
P
If F+S, then
S
F
If U, then P
If R, then N+F
If S, then P
The Big Picture:
Concrete information is much more powerful than the abstract. Always ask yourself,
and focus on, what you do know, instead of what you dont know.
Remember the contrapositive. Dont forget the contrapositive. Always recall the
contrapositive. Never neglect the contrapositive.
Its very possible that you would have been best served leaving this game until last
many test-takers who took control of the section did just that.
LSAT PREP ________________________________________________________________ LSAT Test III Explained: Section I
12 K A P L A N
The Questions:
14. (A)
If sports cars are on exactly 2 floors, then family cars must be on exactly 1 floor, and Rule 1
applies: Sports cars must be on floors 2 and 3, which eliminates choices (C) and (E), and
family cars must be on floor 1, which contradicts choice (B). Floors 2 and 3 contain sports
cars which are production models (Rule 3). Choice (D) is therefore out. Only choice (A),
research models on floor 1, is possible.
Actively work with any new information given in a question. Hearken back to the
rules armed with this new information, and the answer (and the point) will be
yours.
15. (D)
No new information here, so move right on to the choices. Rule 2 kills choices (B), (C) and
(E), since we know that at least one floor has used cars. Rule 3 axes choice (A). Were left
with choice (D), and it is indeed possible that exactly one floor has research models (as
long as that floor is floor 1 since it would have to contain new research model family cars).
When youre given no new information, and the answer is not readily apparent, try
treating the question like an acceptability questioncompare each rule against each
answer choice. In such cases, eliminating the bad choices is just as effective a way
of getting to the right answer.
16. (D)
Another non-if, this one a must be true with no new information. A quick scan through
the choices in search of one of our earlier deductions turns up choice (D)production
models on floor 3. (A), (B), and (C) are possible only, while (E) is totally impossible.
Concrete key deductions will nearly always be tested. If you deduce something,
keep it in the back (or front) of your mind. It will almost always lead to at least one
ten-second questionusually a must be true non-if question like this.
17. (E)
Since we deduced that floor 3 contains production models, the 2 floors with research
models must be floors 1 and 2. A quick glance at your scratchwork tells you that these
research cars must be new family cars (Rules 2 and 3). Notice that this takes care of eight
out of the nine possible boxes in our table. Also notice that the question is a can be false
question. Heres where the on-the-ball test-taker realizes that its a good bet that the answer
to the question will almost certainly deal with the one box thats still unresolved, which
deals with the family/sports issue on floor 3 (everything else is determined, and therefore
must be true). Choice (E), family cars on 3, could be false, and therefore does the trick.
Choices (A) through (D) correspond perfectly to the situation, so naturally they all must
be true.
LSAT PREP ________________________________________________________________ LSAT Test III Explained: Section I
K A P L A N 13
Be active in your pursuit of answersuse all the clues the testmakers give you,
including their choice of question stem. We were able to determine so much in this
question that upon seeing the can be false question stem, it just makes sense to
focus in on the one indeterminate aspect of the situation. This is another example of
how to take control of the test.
18. (D)
Rule 3: All research cars are family cars. Stem: All new cars are research cars. Therefore:
All new cars are family cars, which happens to be choice (D).
Family cars need not be anything in particular, so choices (A) through (C) are out.
Similarly, in (E), a production model car could be a sports car.
Whenever youre given new information in a question stem that doesnt directly lead
to a new deduction, go back and carefully reconsider each of your original
deductions in light of this new information.
Pay attention to the nature of the answer choices; often, this can help inform you as
to the best approach to the question. This one, for example, is the only question
thats not concerned with whats on what floor, but rather deals in generality instead.
This tips us off that the answer will most likely be derived by combining the new
rule in the question stem with one of the rules in the intro.
19. (A)
If all production models are used, then a car that is not used (new), must not be production
(research). So floor 1s new cars must be of the research variety, which means that floor 1s
cars must also be family cars (Rule 3). We get this in choice (A).
(B) through (E) all could be true, but also could be false.
Get in the habit of working with the contrapositive. Whenever youre given new
information thats easily translated into if-then form, the contrapositive should be
one of the first places you visit.
Always take information as far as you can. When youre able to uncover new
deductions based on the hypothetical, do a quick intelligent scan of the choices.
This will help you avoid working through time- consuming choices that are
included specifically to slow you down.
Confidence is key in Logic Games, and LG answer choices are objectively correct.
When you find an answer that youre comfortable with, even if its choice (A), have
the confidence to mark it down and move on. You should only use the other choices
to confirm your choice if youre very unsure of your answer. Realizing that you
dont have to laboriously work through every choice in every question will help you
get in the habit of moving quickly through the section.
LSAT PREP ________________________________________________________________ LSAT Test III Explained: Section I
14 K A P L A N
GAME 4Pilots and Copilots
(Q. 20-24)
The Action: In this grouping game, we're asked to distribute six entitiesthree pilots and
three copilotsamong four planesplanes 1, 2, 3, and 4. While the numbering of the
planes may suggest sequencing, your overview should have dispelled that misconception
right away: There's no mention that these planes are "in a row;" they're not numbered "from
left to right;" and none of the rules say anything about people being "in adjacent planes," or
anything like that. Our job is simply to distribute the pilots and copilots into the planes.
The main Key Issue, therefore, is a grouping concern:
1) Whos in what plane? And by extension: Which pilots and copilots can, must, or cannot
fly in the same plane as which other pilots and copilots?
The Initial Setup: Keep this setup simple; four circles or columns, numbered 1 to 4, can
represent the planes. Then list the pilots and copilots off to the side:
1 2 3 4
P C
ABC DEF
The Rules:
1) and 2) You most likely already used these rules to get a handle on the entities. However,
some test takers overlooked a key element of these rulesthat the pilots and copilots are
all aboard planes that are flying in the show. This means that everyone flies. Selecting who flies
isnt an issue; theyre all up in the air. The only question here is which plane each person is
in. As for listing the entities, some find it helpful to use capital letters for the pilots and
lowercase letters for the copilots, some dont. Do whats easiest for you.
3) Translation: Every plane thats flying needs one pilotat least one. This rule says
nothing about copilots, nor does it imply that only one pilot may fly in a particular plane.
So far, its quite possible that exactly one pilot flies a plane without a copilot, just as its
possible for more than one pilot to fly in one plane.
4) This rule is basically a loophole closer to ensure that no one from the audience or
anywhere else rushes out and pilots a plane.
LSAT PREP ________________________________________________________________ LSAT Test III Explained: Section I
K A P L A N 15
5) Anna will only fly in plane 1 or plane 4. Since everyone is flying in the show, we know
that one of these planes must be used. To build this into our master sketch, write A with
arrows pointing to planes 1 and 4.
6) Dave only flies in plane 2 or plane 3 D with arrows to 2 and 3 takes care of this.
Key Deductions: Not much in the way of deductions, but there are a few issues that are
worth working out before hitting the questions. First, the numbers: No plane flies without a
qualified pilot aboard. But we have only three qualified pilots, which means that a
maximum of three of the four planes are flying; at least one is going to remain empty
and on the ground. Also, since a plane cant fly without a pilot (Rule 3), we know that one
of the pilots must join Dave (a copilot) in either plane 2 or plane 3. It cant be Anna, since
shes in plane 1 or 4, so Dave must fly with either B or C, in plane 2 or 3.
The Final Visualization: Heres what were armed with to reel in these five questions:
1 2 3 4
At Least
1 Pilot
Either B or C
with D
A
D
The Big Picture:
Dont forget that critical reading is incredibly important for Logic Games as well as
for the other sections of the test. Sometimes, the testmakers only implyi.e., dont
clearly spell outinformation that proves to be vital to the game. Interrogate the
stimulus: Do all pilots fly? Yes. Do all copilots fly? Yes. Do all planes fly? No
in fact thats impossible. As many planes fly as are needed to get the 7 people aloft.
Attempt to weed out the games major concern. Not every game has one specific
major concern, but in the ones that do, focusing on this aspect will help you in
almost every question. In this case, our major concern is who will accompany
Dave? At least Bob or Cindyif not bothmust do so, so you should find it
helpful to incorporate new information from the question stems into the context of
how it relates to taking care of Dave.
Use the hints provided for you in the question stems. The phrase in Q. 23s stem If
plane 1 is used clearly implies the possibility of cases in which it is not used. True,
this question comes towards the end of the game, but very often in a game whose
action is slightly unusual or complex, one of the first few question stems will help to
clarify the situation.
LSAT PREP ________________________________________________________________ LSAT Test III Explained: Section I
16 K A P L A N
The Questions:
20. (B)
Anna is in plane 4 and Dave is in plane 2. Well, we knew wed have to focus on the Dave
situation, and here it is right off the bat. Since Bob or Cindy (or both) needs to accompany
Dave, Cindy in 3 forces Bob into 2 with Dave.
(A) No reason why Cindy couldnt fly in plane 2 with Dave.
(C) No, Bob could fly in plane 2.
(D) No; if Bob flies in plane 4, Cindy would have to fly with Dave in plane 2.
(E) If Cindy flies in plane 2 with Dave, then Bob could fly in any of the planes.
Focus on the games key issues and big concerns. Asking yourself: How do I take
care of Dave? leads to this answer, even though the correct choice is in if-then form.
If youve considered the new information and still nothing comes to mind, trying
out each choice is a much better use of your time than staring at the page.
21. (C)
Anna cant fly with Dave (Rules 5 and 6). If Bob joins Anna, he cant fly with Dave. So
Cindy would have to fly with copilot Dave, choice (C). Choices (A), (B), (D), and (E) all
could be true, but none of them must be true.
Dont be surprised when the testmakers use one or two basic concepts as the key to
the majority of the questions (like running out of pilots to place with Dave). The
sooner you can recognize this MVC (Most Valuable Concept), the better.
22. (D)
Cindy and Fran fly alone, Anna never flies with Dave, so Bob must fly with Dave, choice
(D). (A) is dead wrong, and (B), (C), and (E) are merely possible.
Unless youre unsure of the workings of the game and you need to test you thinking,
you should never continue checking the choices after youve found the answer.
Trust your work, and save yourself some precious time.
LSAT PREP ________________________________________________________________ LSAT Test III Explained: Section I
K A P L A N 17
23. (B)
This is very much like a standard acceptability question. (D) and (E) bite the dust thanks to
Rule 6. Dave cant fly in 1 but must have a pilot with him in plane 2 or 3, so any
prospective plane-1 crew that included all three pilots would be impossible. That
eliminates Choice (A). (C) leaves only Anna to join Dave which isnt possible. That leaves
us with (B)Anna, Bob, Ed, and Frana perfectly acceptable crew for plane 1, with Cindy
and Dave in plane 2 or 3.
It is important to learn to recognize acceptability questions even when they are not
structured like normal acceptability questions. These questions are among the
easiest on the test. If you were running short on time, a quick scan of the questions
should have suggested starting with this one (which also helps to clarify the action
of the game, as discussed above).
24. (C)
Dave and one pilot cant be in 4, but everybody else can be. That's the maximum, four
people, choice (C).
When looking for maximums, start with finding the entities that CANNOT be
included. Similarly, when asked for a minimum, start looking for entities that MUST
be included. This is much quicker than the other way around.
This game may exaggerate the point, and weve certainly harped on it here, but some
games are based around central elements, so much so that these element plays a big
part in every single question. This game was all about getting a pilot to go along with
Dave in 2 or 3. The games youll see on your test will be based around other
dominant issues. Attempt to seek them out; youll be doing yourself a great favor.
18 K A P L A N
SECTION II:
LOGICAL REASONING
LSAT PREP ________________________________________________________________ LSAT Test III Explained: Section II
K A P L A N 19
1. (C)
In this parallel reasoning question, the stimulus is fairly easily to symbolize: If you
have a lot of money in the bank, (if X), your spending power is great (then Y). If your
spending power is great, (if Y), then you are happy (then Z). The conclusion
combines the first two sentences: If you have a large amount of money in the bank, (if
X), then you are happy, (then Z). So the complete argument in symbols would read
``If X, then Y. If Y, then Z. Therefore, if X, then Z. Choice (C) has the same form as the
stimulus argument: If you swim, (if X), then your heart rate increases (then Y). If Y,
then you are overexcited (then Z). Therefore, if X, then Z.
(A) If you have good health, (if X), then you can earn a lot, (then Y). If Y, then you
can buy an expensive house, (then Z). Therefore, if X, then you can have a
comfortable life, (then Q?)
(B) If you drink too much, (if X), then you will feel sick, (then Y). If X, then you will
have no money left, (then Z). Therefore, if Z, then Y.
(D) If you exercise, (if X), then you are fit, (then Y). If X, then you are exhausted,
(then Z.) Therefore, if Y, then Z.
(E) If you have a lot of money in the bank, (if X), then you are confident about the
future, (then Y). If you are optimistic, (if Z), then Y. Therefore, if X, then Z.
On parallel reasoning questions, try to symbolize the argument whenever
possible.
Be careful of choices like (A) and (E), in which the content is quite similar to
that of the stimulus. Content has nothing to do with the form of the reasoning.
And the form is what were asked to recognize here.
2. (E)
The stimulus concludes that a persons birth sign influences their personality. We
want to cast doubt on the conclusion that the subjects, because they were Geminis,
were more sociable and outgoing than the average person. The key here is that the
subjects were all volunteers. Would shy, introverted people offer to appear on a TV
program? Probably not. (E) picks up on this, offering a plausible explanation which
suggests that the method itself, and not astrology, could have produced the results.
(A) is an au contraire choice. Since the test results matched the investigators
impressions, the test would seem to be more valid if it were administered by
impartial people.
(B) is useless background information; since astrologers are far from impartial in this
discussion, their claims cannot be used as plausible evidence. In any case, to say that
non-Geminis are less sociable than Geminis supports the investigators case, so this
is also an au contraire choice.
LSAT PREP ________________________________________________________________ LSAT Test III Explained: Section II
20 K A P L A N
(C) also seems to be supporting the work of the investigators, rather than casting
doubt upon it. If the investigators first impressions of people are confirmed by their
later observations of the people, then all this proves is that the investigators are good
observers and good judges of character, right? This certainly doesnt indicate a flaw
in their method.
(D) is more useless background info. Whether its likely that there are more Geminis
on the street than in the general population has little to do with the possible
influence of astrology on personality and in no way criticizes the investigators
research methods.
Make the question stem work for you. Here, were told point blank that there
is a flaw in the investigators method; all we have to do is find it.
On questions that involve surveys, be alert for clues that the survey sample is
or is not representative of the population about whom the conclusion is
drawn.
3. (D)
The conclusion: North American children can be made physically fit only if they
have daily calisthenics at school. The evidence: European children, who engage in
calisthenics each day at school, are stronger, faster, and less easily-winded than
North American children, whose schools rarely offer daily calisthenics programs.
The assumption: calisthenics are a key part of the European childrens physical
fitness. Choice (D) neatly paraphrases this assumption, and is the correct choice.
(A) confuses necessity with sufficiency. The author assumes that calisthenics are
necessary to insure physical fitness; whether they are sufficient to do so is a different
question.
(B) This need not be so in order for the conclusion to remain valid; try the Denial
Test, and youll be convinced.
(C) and (E) are beyond the scope of the argument. The author never mentions health
(C) or nutrition (E), so he or she neednt assume anything about either.
The assumption here, that its the calisthenics which are chiefly responsible
for European childrens physical fitness, is so reasonable that you might have
missed it; but its still a necessary assumption.
You can check this one nicely with the Denial Test. If we were to assume that
school calisthenics are not a vital factor to the fitness of European children,
then there must be some other factor which causes European kids to be so fit.
And that would make the conclusion (American kids can become fit only if
they participate in school calisthenics) fall apart.
LSAT PREP ________________________________________________________________ LSAT Test III Explained: Section II
K A P L A N 21
In general, watch out for extreme, unqualified statements in an arguments
conclusion, e.g. the authors recommendation is the only possible solution.
These will often help you identify key assumptions.
4. (B)
If buildings have to be unobtrusive in order to be inviting and functional, and
modern architects produce buildings that are not functional because their strong
personalities take over their work, then we can conclude that these specific
architects are producing buildings that are not unobtrusive.
(A) and (D), much like (A) in the previous question, confuse necessity with
sufficiency. It may be necessary for a building to be unobtrusive (A) and take second
place to the environment (D) in order for it to be inviting and functional, but that
doesnt mean either is sufficient for that to happen.
(C) Simply because an architect has a strong personality, it doesnt mean that he or
she must let that personality take over. We know that in some cases this has been
true, but perhaps some architects can control their strong personalities, and still be
able to produce unobtrusive buildings.
(E) Its never stated that an architect cant put his or her personality into a building
without having it be obtrusive; were told only that architects who let their strong
personalities take over their work havent produced buildings that are functional for
public use.
In questions that ask you to draw a conclusion, be careful not to assume
anything thats not explicitly stated in the argument. Here, we cant assume
that all architects with strong personalities let those personalities take over
their workas in choice (C)or that architects cannot express their
personality in their work without the personality taking overas in choice
(E).
This was a good question for using the strategy of elimination. Even if you
werent sure about the correct choice (B), choices (A) and (D) contain the
common error of confusing necessary and sufficient, while choices (C) and (E)
depend on assumptions that just arent in the passage.
5. (E)
The director argues for the funding of the megatelescope on the grounds that the whole
world benefits from new technology and new inventions, and that funding for these
ventures is not beneficial to only the scientists themselves. The director uses Maxwell,
Newton and Einstein as examples of scientists who were not limited by a lack of
funding, and were, therefore, able to make discoveries that benefited the whole world.
Clearly, the director is drawing an analogy between the megatelescope research and the
research of those three great scientists. Thats a pretty heady comparison to make; the
author needs to present evidence showing that the megatelescope research may
LSAT PREP ________________________________________________________________ LSAT Test III Explained: Section II
22 K A P L A N
approach the same level as that done by these great scientists. (E) is therefore the
strongest criticism of the argument.
(A) The director is using Newton and the others as examples of earlier scientists who
made great discoveries; there is absolutely no appeal to the authority of these long-dead
people on the subject of the megatelescope. And since theyre the only experts
mentioned, (A) isnt a possible criticism of the argument.
(B) is irrelevant. It really doesnt matter who opposes the development or funding of the
megatelescope, because the opponents of this argument arent being attacked by the
director; only their point of view is questioned.
(C) is a distortion. Charging that someones point of view is dangerous is distinctly not
the same as launching a personal attack on that person.
(D) makes an irrelevant distinction. The word ``benefit in either the economic or the
intellectual sense would have the same effect on the argument, since the astronomers,
along with the rest of the world, could reap either or both kinds of benefits from the
funding.
This question is an excellent example of how the LSAT relates to the reasoning
skills required in law school and in the legal profession. Many legal questions
hinge on whether a particular case is analogous to a past case. In court, you cant
just say that this case is analogous to another; you have to provide some evidence.
And thats just what this stimulus fails to do.
6. (C)
The author argues that a fare hike of forty percent must be implemented even though it
will cause economic hardship for users of transportation. The evidence for this
conclusion is that if the fare doesnt increase, service will be cut, and a large loss of
ridership will occur. The author doesnt given any reason why the fare hike should
occur; she only outlines the negative consequences that will result if it doesnt occur. (C),
which states that the author arrives at a conclusion indirectly by rejecting an alternative,
explains her strategy quite clearly.
(A) directly contradicts the stimulus; the author freely admits that some riders will
experience hardship because of the hike.
(B) The argument does explore the other side of the issue, and its consequences, but
theres no indication that a supporter of an alternate position would face a contradiction.
(D) The author doesnt argue by defending her proposal against objections leveled at the
alternative. Rather, she herself raises objections against the alternative and argues that
they are stronger than those that can be raised against her proposal, which is to raise
fares.
LSAT PREP ________________________________________________________________ LSAT Test III Explained: Section II
K A P L A N 23
(E) is out in left field. Theres no mention of past actions with regard to fare
increases, and the author certainly doesnt prove anything by using evidence from
the past.
For method of argument questions, eliminate immediately choices such as (A)
and (E) above that are blatantly inconsistent with the passage.
When the answer choices are in an abstract form, you may find it easier to try
to fit each choice to the argument, eliminating those choices that just dont
work.
7. (A)
Everyone who participates in local politics has an influence on the communitys
values. Since some of those people are selfish opportunists, we can conclude that
some selfish opportunists have an influence on the communitys values.
(B), (C), (D), and (E) could be true. None of these must be true.
This question offers a great example of how the testmakers test formal logic.
Most of the time in formal logic situations, youre called upon to put two and
two together, so use your Logic Games skills of combining rules and making
deductions.
When the question asks you to draw a conclusion, be aware of the difference
between what must be true (follows without question from the evidence), what
could be true (is not contradicted by the evidence), and what cannot be true (is
directly contradicted by the evidence).
Note the precise use of language on the LSAT: The first sentence leaves open
the possibility of someone participating in local politics who is neither
interested in public service nor a selfish opportunist. Thats why (E) doesnt
have to be true.
8. (B)
The discrepancy: lighteners, which are without cholesterol, raise the blood
cholesterol levels of consumers higher than does the milk, which contains 2
milligrams of cholesterol. The key here is that lighteners contain more saturated fat
than milk. So were looking for a choice that will explain the relationship between
saturated fat and cholesterol, with regard to blood cholesterol levels. (B) does just
that.
(A) is useless background information. The nutritionists recommendation doesnt
explain why a product which doesnt contain cholesterol, like a lightener, would
produce more blood cholesterol than a product like milk, which does contain
cholesterol.
LSAT PREP ________________________________________________________________ LSAT Test III Explained: Section II
24 K A P L A N
(C) adds an irrelevant distinction. Light cream has absolutely no bearing on the
issue, which is the relative effects of lighteners with no cholesterol, and milk with
cholesterol.
(D) is irrelevant, because it brings up a lightener that doesnt contain coconut oil. So
what if this type of lightener has less fat and cholesterol than milk? It still doesnt
help to resolve the discrepancy involving coconut oil lighteners.
(E) This choice explains the relationship between the fat and cholesterol levels of
most dairy products, but it doesnt address the difference between the two different
products that form the basis of the paradoxthe non-dairy lightener with high fat,
and the dairy product (milk with cholesterol).
Even if you were overwhelmed by the confusion of terms and measurements
like cholesterol and blood cholesterol and grams and milligrams, you should
have been able to arrive at the correct answer by process of elimination. (B) is
the only one that deals directly with the relative effects of cholesterol and fat
on blood cholesterol, so its the only one that can possibly resolve the
discrepancy.
The first step in a paradox question is to define the paradoxyou cant
possibly find a good explanation if youre not sure what youre trying to
explain.
9. (D)
If the consumer is using a very small amount of lightener, as opposed to a very large
amount of milk, then it follows that the amount of cholesterol in the large quantity of
milk will add up and have a greater effect on the consumers blood cholesterol than
the amount of saturated fat in the smaller quantity of lightener. So, if (D) is true, the
manufacturers claims would be considerably strengthened.
(A) neither weakens nor strengthens the argument. Were talking about changes in
the typical consumers levels, changes due to lightener or milk. The effect of health
practices on some peoples cholesterol levels is beyond the scope of the argument.
(B) The desserts that accompany coffee have nothing to do with the effect of
lightener, as opposed to milk, on consumers cholesterol levels.
(C) is a useless distinction. Coffee lighteners that are not based on coconut oil are
irrelevant to the claims of manufacturers of lighteners that are based on coconut oil.
(E) Dismissing the possibility of psychosomatic effects, the beliefs of the consumer
really dont enter into this discussion.
Milk (no pun intended) the two-question stimuli for all its worth: Heres a
situation in which the two questions for a single stimulus really feed off each
other. That is, the work you did on Question 8 should put you in a better
position to answer Question 9.
LSAT PREP ________________________________________________________________ LSAT Test III Explained: Section II
K A P L A N 25
While it may look intimidating, a lengthy question stem that adds new
information need not make for a killer question. Its like a Logic Games
question stem that adds new information in the hypothetical; its unusual in
Logical Reasoning, but manageable nonetheless.
10. (E)
This is really a thinly veiled formal logic stimulus. The first sentence can be put into
if-then form: If people have serious financial problems, then they cant be happy.
And this, of course, is logically equivalent to its contrapositive: if people are happy,
then they do not have serious financial problems.
(A) and (D) We cant infer that serious financial problems (or in (A)s case, serious
problemsnotice the scope shift) are the only things that can make people unhappy.
In this case, serious financial problems are sufficient to make people unhappy, but
that doesnt mean that this condition is necessary for unhappiness.
(B) and (C) also confuse necessary and sufficient conditions. Notice that when we
negate the conditions, the paradigm shifts: Not having serious financial problems is
necessary to being happy (according to the passage), but it is not sufficient.
You could have eliminated (B) and (C) if you recognized that they are
contrapositives of each other, and therefore logically equivalent. Since each
question has only one right answer, both (B) and (C) must be wrong.
Use the test to your advantage: Some sections harp on the same concepts over
and over again. To this point, the concept of necessity vs. sufficiency has
appeared in some form or another in a handful of stimuli and answer choices.
11. (D)
This ones a method of argument question, so the operative question is: whats the
author doing? Hes presenting a belief, or proposal, and then telling us why, if the
proposal were put in practice, it would have illogical, foolish results. (D) is the
closest paraphrase of this.
(A) is a clear contradiction of what the author is trying to do. True, he is stating a
general principle, but hes presenting an argument against, not for, adopting it.
(B) The author doesnt offer specific evidence of unfavorable consequences that
have occurredhe offers his view about what would happen if the principle in the
first sentence were adopted, but never provides actual results of real applications of
the principle.
(C) distorts the authors main point. The thrust of the argument is not that the
expected consequences wont result, but rather that unexpected ones will.
(E) The author is upset about what would happen if the principle were applied.
Whether the principle can be uniformly applied, or applied at all, is not in question.
LSAT PREP ________________________________________________________________ LSAT Test III Explained: Section II
26 K A P L A N
In method of argument questions, the correct choice must be fully correct.
Read each choice meticulously, and dont be fooled by choices like (A), which
is only half right.
12. (D)
The evidence: photovoltaic power plants, which produce electricity from sunlight,
are now one-tenth as costly as they were twenty years ago, whereas traditional power
plants have increased in cost. The conclusion: photovoltaic plants produce
electricity less expensively than do traditional plants. But the fact that one method is
cheaper than it used to be while another is more costly than it used to be is not
enough to conclude that the first is therefore cheaper than the second today. We
need some link between the costs of the two methods that will allow the conclusion
to stand, and the assumption in (E) does the trick.
(A) simply restates some of the evidence.
(B) makes an irrelevant distinction. The amount of electric power is not addressed in
the stimulus; its the cost of producing the power that were concerned with here.
(C) The author neednt assume that none of the advances can be applied to
traditional plants in order to conclude that photovoltaic plants produce electricity
less expensively than do traditional plants. The main issue is the relative cost of the
two methods, and since this doesnt tie directly into that, it isnt relied upon by the
stimulus.
(E) So what? Unless we know what the cost of a traditional plant is compared to that
of a photovoltaic plant, we cant say that the argument is properly drawn.
Recognize your strengths and weaknesses. If numerical/statistical situations
arent your cup of tea, dont fight with the questionmove on to friendlier
ground and come back to this one at the end if time permits.
Be wary when an author gives separate evidence about two groups and then
offers up a conclusion that attempts to compare the groups.
13. (A)
Heres another parallel reasoning stimulus that lends itself to symbolic
representation: If X (that insect is a bee), then Y (it can only sting once). Y (it only
stung once), therefore X (it is a bee). This faulty structure is matched by choice (A). It
takes some rearranging, but the elements of (A) boil down to: If X (it is spring), then
Y (I cannot stop sneezing). Y (I sneezed), therefore X (spring is here). Note how it
exhibits the same flaw as the original: In the stimulus, its very possible that the
insect is not a bee, and is another insect that just happened to sting only once. In (A),
its possible that its not spring, and that the person in question just happened to
sneeze during some other season.
LSAT PREP ________________________________________________________________ LSAT Test III Explained: Section II
K A P L A N 27
(B) If X (the sky is clear), then Y (the atmospheric pressure is high). Almost X (its
clearing up), therefore almost Y (the pressure is bound to be high soon).
(C) If X (the painting is old and brittle), then Y (it will be moved with extreme care).
Not Y (that painting is not moved with extreme care), therefore Not X (it is not old
and brittle).
(D) If X (there was one more thunderstorm), then Y (the roof would be ruined). Not Y
(the roof is fine), therefore Not X (there must not have been any thunderstorms).
(E) Really doesnt fit the structure weve set up. We might be able to say: If X (one
survives in the wild), then Y (one has physical stamina like Marks). But then we get
into Marks fear of spiders (Z?).
In parallel reasoning questions, its sometimes necessary to rearrange the
terms of the argument in order to identify the correct choice. Remember, were
looking for a parallel logical structure, not a parallel verbal structure.
Remember your goal. Were not asked to find a logically valid argument. Both
(C) and (D) are logically correct, and therefore neither is parallel to the faulty
argument in the stimulus.
14. (B)
Quincys argument is that physician training does not need to change because it has
worked in the past. To counter this, we need a choice that shows that current
medical practice is somehow different than in the past, and therefore requires a
change in training methods. Choice (B) fits this qualification nicely. If (B) were true,
it would mean that physicians in training would have to deal with more crisis
situations than did physicians in the past, and therefore, that the mental and
emotional strain of the long work hours may make them more likely to make faulty
decisions.
(A) is an au contraire choice. If the responsibilities of the resident staff have not
changed over the past decades, then Quincy seems right in arguing if it aint broke,
dont fix it.
(C) involves a scope shift: Were concerned with medical practice in general, not ER
patients especially.
(D) makes a useless distinction. Were not concerned with the differences in
workload among the different specialties.
(E) doesnt give us enough information to effectively counter Quincys argument.
Although it argues for observation over thirty-six hours, and therefore could
support the need for residents to be on rounds for that long a period, were not told
that the observation needs to be continuous.
LSAT PREP ________________________________________________________________ LSAT Test III Explained: Section II
28 K A P L A N
In dialogue questions, be sure to remember whose argument you are trying to
support or weaken.
In this question, you needed to read Pamelas argument for background. In a
pinch, however, you could have eliminated (A) and (E) just from reading
Quincys side.
15. (D)
Experiments like this one are logically valid only if the two groups are exactly alike
to begin with and if one of them is exposed to one variable. In this case, the variable
is being shown violent TV programs right before play. Since the author has
concluded that the experiment was valid, she is assuming that the two groups had
no differences other than the exposure to violent TV programs; in other words, that
the television programs were the sole cause of the violence, and that nothing else
could have been the cause. Choice (D) correctly identifies this assumption.
(A) is a scope shift. The author is not talking about the effect of all television
programs on all of society. The focus of this passage is the effects of violent
television programs on children.
(B) What if theyre not? Ultimate responsibility isnt the issue. In fact, the
arguments thrust is to show the responsibility of TV, not parents.
(C) is an au contraire choice. In order to make her argument, the author must believe
that violence and passive observation of violence (in this case, watching violence on
television), are directly related.
(E) is beyond the scope. The author never mentions any violent treatment toward the
children.
In arguments involving the interpretation of experimental results, the
conclusion often depends on the assumption that there were no unaccounted-
for factors that could have caused the results observed.
This assumption was so obvious you might even have missed it, thinking the
author wouldnt have missed that. Dont take for granted anything about the
author or the argument that isnt in the text.
16. (C)
Because waste gets disposed of in less populated areas, those who are responsible
for dumping are not as fearless about its effects as they claim. This assumes that
there is no plausible alternate explanation for the disposal pattern. (C) weakens the
argument by giving a reasonable alternative explanation: Dumping nuclear waste in
less populated areas poses fewer economic and bureaucratic problems than
dumping in areas of denser population.
LSAT PREP ________________________________________________________________ LSAT Test III Explained: Section II
K A P L A N 29
(A) and (B) are au contraire choices. The acknowledgment that there could be an
accident indicates that nuclear waste does, in fact, pose some threat to people.
(D) is an irrelevant comparison. Pointing out chemical dangers wont show that
nuclear waste is safe; theyre unrelated.
(E) supports the authors argument. It seems to be a statement of what the policy
makers really believe, but just wont admit.
To weaken an argument, its not necessary to disprove it completely. We
simply need to make it less likely for the conclusion to follow directly from
the evidence.
17. (C)
The United States has, overall, seen a decline in its infant mortality rate in the past
few years. But this does not mean that the babies born in the United States are
healthier now than they were in the past. So the author is assuming the existence of
an alternate explanation for the decline in the infant mortality rate. To support the
argument, we need a choice that offers this alternate explanation. Choice (C) tells us
that the United States has developed technology that can save babies that would
have died otherwise. So while we may have just as many sick or premature babies
being born, we have fewer babies dying as a result of sickness or premature birth
thanks to the advanced technology. This would explain why the decrease in
mortality rates has no connection with the average health of the infants.
(A) is useless background information; it doesnt address the contrast between the
overall infant mortality rate and infant health.
(B) doesnt help us support the authors claim; we dont need more information
about infant death. What we do need is information that would explain why a
decline in the infant mortality rate doesnt signal an increase in health.
(D) Weve already been told that the overall infant mortality rate has been declining.
Like (A), this provides background, but doesnt do anything to explain or support
the claim that overall infant health hasnt improved.
(E) is a left-field choice. Were concerned with why health hasnt improved along
with the infant mortality rate; were not interested in the relationship between babies
health and how much attention they receive.
Be sure to keep track of the argument that youre attempting to manipulate. In
this question, the authors argument is that another argument (connecting
decreasing infant mortality with increasing infant health) is flawed. So to
support the author, were actually trying to weaken the first argument.
LSAT PREP ________________________________________________________________ LSAT Test III Explained: Section II
30 K A P L A N
18. (C)
The authors whole point is that Raghnalls conclusion is based on inadequate
evidence. The authors evidence is the alternative explanation he provides for the
surveys results; namely, that couples may blame finances for their marriage
problems when finances arent the real problem. He uses this alternative explanation
to make the point that Raghnall has jumped to conclusionsthat she has failed to
consider other possible explanations for the surveys results. Thus, the author
believes that Raghnalls conclusion is inadequately justified.
(A) distorts the argument. The authors point isnt so much that financial problems
are not a big factor in the breakup of marriages, but rather that Raghnall cannot
reasonably conclude that they are without additional evidence.
(B) is outside the scope, a sure sign its not the main point. Marriage counselors have
never even been mentioned.
(D) simply restates the evidence, and not even the authors evidence, but Raghnalls.
(E) is a subtle misreading. The author does allude to a number of other articles,
but all we know is that these articles relied on the same survey that Raghnalls did; we
dont know that they necessarily drew the same conclusion that Raghnall did.
The answer to a main point question will usually be a paraphrase of the
authors conclusion. Anything else can be eliminated.
When an argument involves a critique of somebody else, be sure to keep the
authors evidence and conclusion separate in your mind from the evidence
and conclusion being critiqued.
19. (B)
We pretty much answered this in our analysis of 18: The author offers a different
interpretation of the surveys resultsbasically, that couples often express their
frustrations about other aspects of their marriage in financial terms, blaming money
when money isnt the problem. In other words, the author undermines Raghnalls
conclusion by offering an alternative explanation for some of the data on which his
conclusion was based.
(A) The author never supplies us with a specific counterexample. He gives us an
alternative explanation, but he never gives us any specific examples, such as:
Couple X blamed money for their problems, but their real problem was...
(C) is off-base because all the author claims is that the survey doesnt establish that
financial problems are the major problem in marriages. Financial problems or
`money can hardly be called an emotion, and Raghnall hasnt referred to anything
else as a cause of divorce.
(D) shifts the scope a bit: The author never criticizes the survey; he criticizes
Raghnalls conclusion, which is based on the survey.
LSAT PREP ________________________________________________________________ LSAT Test III Explained: Section II
K A P L A N 31
(E) goes too far. The author never demonstrates or shows that couples cannot
accurately describe their marital problems. He merely claims that they often fail to
do so.
Scanning the answer choices, we can see that all five choices begin with the
words undermines a conclusion and that 4 choices begin undermines a
conclusion drawn from statistical evidence by. Save time by focusing your
attention on the latter part of each choice.
In cases where two questions are drawn from the same stimulus, you can save
time by using your analysis from the first question to help you get the second.
For this reason, try not to skip double-question stimuliwhen handled well,
its like getting two points for the price of one.
20. (C)
Since were asked to pick the one choice out of five that does NOT weaken the
argument, we know that the argument will be pretty vulnerable to weakeners. There
are a number of assumptions at work in this argument, many of which lead to the
weakeners in the wrong choices, but choice (C) is correct because its totally
irrelevant to the issue of honey production.
(A) weakens the argument by undermining the major assumption that whats true of
Brazil will be true of the U.S. If, as (A) claims, the native bees in Brazil are different
from the ones here, then the comparison the author cites is irrelevantmaybe
domestic U.S. bees produce more honey than both Brazilian and Africanized bees
do.
(B) and (D) both undermine another one of the arguments basic assumptions: that
commercial honey production wont decline for some other reason. If, as (B) says, its
more costly and difficult to use Africanized bees, or if, as (D) says, a lot of the
people now responsible for honey production would rather cease and desist than
use Africanized bees, then its quite possible that commercial honey production will
decline if these bees are introduced. In any case, the author can no longer conclude
for sure that it wont decline.
(E) If Africanized bees are better suited to Brazil, then the author can no longer
assume that in America, theyll produce more honey than American honeybees.
Who knows, maybe theyll all die off once they get here.
For all EXCEPT questions, elimination is often the best strategy; theres so
much going on, that you probably shouldnt worry too much about pre-
phrasing an answer. If you have a solid grasp of the argument, chances are
youll know the correct choice when you see it.
LSAT PREP ________________________________________________________________ LSAT Test III Explained: Section II
32 K A P L A N
21. (B)
According to a recent report, low blood cholesterol weakens artery walls, increasing
the likelihood that the arteries will rupture, and thereby bring about a cerebral
hemorrhage. The author concludes that this new report supports the long-held
belief of Japanese researchers that Western diets are better at protecting against
cerebral hemorrhage than are non-Western diets.
For this conclusion to be valid, the author must be assuming that Western diets lead
to a higher blood cholesterol level than non-Western diets.
(A) A healthier diet isnt the issue herewe need information that fills in the
connection between the blood cholesterol evidence and the conclusion of the
Japanese researchers in the last sentence, and this isnt it.
(C) The author tells us that high blood cholesterol lowers the risk of weakened artery
walls. He never says that it eliminates this risk, nor does his argument depend on this
information.
(D) is an irrelevant comparison. In fact, the author could assume that cerebral
hemorrhages are less dangerous than strokes caused by blood clots, and it wouldnt
damage his claim that Western diets are less likely to lead to cerebral hemorrhages.
(E) involves a scope shift: Low blood pressure is an irrelevant issue, because we dont
know how this relates to blood cholesterol.
Dont be intimidated by scientific language or technical terms. You didnt have
to know anything about medicine to answer this question. In fact, if you did
know that blood pressure is often related to cholesterol levels, that would have
just gotten you into trouble, since blood pressure isnt mentioned in the
argumentbut is dangled as a red herring in wrong choice (E).
22. (A)
This is simply a matter of a very common chain of argument: IF X, then Y. If Y, then
Z. Therefore, if X, then Z. Specifically: If the country is to remain internationally
competitive, then there is an undeniable need for citizens to better understand
international affairs. If theres a need for this better understanding of international
affairs, then all of our new teachers must be prepared to teach their subject matter
with an international orientation. This allows us to infer that the first statement leads
to the third: If the country is to remain internationally competitive, then all of our
new teachers must be prepared to teach their subjects with an international
orientation.
(B) and (C), like many choices before them on this test, confuse necessary and
sufficient conditions. The stimulus establishes the ability of new teachers to teach
with an international focus as necessary for the country to remain competitive. We
cant infer, as (B) suggests, that its sufficient for this. And as for (C), better
understanding of international affairs by our citizens is also necessary, but not
sufficient, for the country to remain competitive.
LSAT PREP ________________________________________________________________ LSAT Test III Explained: Section II
K A P L A N 33
(D) completely denies the authors first if-then statement, which is an important
premise in the argument.
(E) All were told about public reports is that they stressed the need for citizens to
better understand international affairs. We have no evidence that they said anything
about training teachers to teach with an international focusthat requirement was
inserted by the author.
The presence of a relatively simple question this late in the section is a good
reminder to keep moving and make sure you get a chance to attempt every
question.
23. (C)
Were asked to weaken the proponents argument, and conveniently enough, were
given their assumption. Your best bet is to look for a choice that undermines that
assumption, which we get in (C): If genetic characteristics can occur in sets, that
breaks down the proponents assumption that these characteristics occur
independently.
(A) is an au contraire choice. If the genetic material that all people have in common
with each other and with animals is excluded from the procedure, then that makes
the analysis much more specific, and, inferably, more valid.
(B) If anything, this choice strengthens the proponents claim by dismissing a
possible argument against the procedure; the argument that its unreliable because
different people could obtain different readings of the same pattern.
(D) contains a scope shift: The point here is whether the procedure is valid when done
accurately.
(E) is irrelevant. The issue is whether or not DNA fingerprinting can be accurately
used to match two different samples of genetic material. Any other use is a separate
issue.
Use the information youre given. Here, were explicitly told that proponents of
DNA fingerprinting base their claim on an assumption of independent
occurrence between genetic characteristics. So all we have to do to weaken the
proponents claim is to weaken that assumption.
LSAT PREP ________________________________________________________________ LSAT Test III Explained: Section II
34 K A P L A N
24. (A)
The argument compares schools to cultures; schools would have to progress the
same way cultures progress. Cultures cannot progress if outsiders impose their
views. So the same must be true for an individual school: If a school is to progress,
it must be free of outside imposition. Choice (A) carries out this analogy between
schools and cultures.
(B) The author doesnt mention degrees of independence, or degrees of initiative; he
merely claims that the only way a culture can progress is if it is independent of
outside imposition.
(C) Were told that progress for an individual school, or culture, requires
independence from outside imposition. The idea of school system officials deciding
what changes to make is directly counter to this.
(D) goes too far. The author never says outsiders must be totally shut out if progress
is to be achieved; in fact, he admits they can provide valuable advice.
(E) A similar story: this choice interprets independence as sufficient for progress,
when in fact, all weve been told is that its necessary. Certainly, there are other
factors that could come into play.
Complete the passage (also known as fill-in-the-blank) questions are
simply another way in which the LSAT asks you to draw an inference based
on the information provided.
The answer to a complete the passage question must be consistent with the
authors main idea and tone. Choices that are clearly inconsistent with the
main idea can be eliminated immediately.
LSAT PREP ________________________________________________________________ LSAT Test III Explained: Section II
K A P L A N 35
25. (E)
The key here is the authors vague use of the term adequate. If our main enemy is
gone, whos to say that a reduced defense budget wouldnt be adequate? If the
primary motivation for spending so much money on defense in the first place is no
longer relevant, isnt it reasonable to expect that we could spend less money and still
defend ourselves as adequately as before?
(A) distorts the passagesince the whole point is that the public may no longer be
persuaded, the author certainly doesnt argue that this manipulation can continue
indefinitely.
(B) just plain denies what the author says, which does not amount to finding a
weakness in his reasoning. We want a criticism of his reasoning, not merely an
unsupported denial of his facts.
(C) is a fancy way of accusing the author of using circular reasoning. However, the
evidence (the dissolution of the Eastern bloc) is quite different from the conclusion,
which is that there will be insufficient support for an adequate defense budget.
(D) The author does give a reason for his opinion: the dissolution of the Eastern bloc.
It may not be a very good reason, but its a reason all the same.
When an argument compares past and present conditions, a common flaw is
the assumption that certain terms or descriptions can be applied equally to
both periods. When an author documents a changing situation, make sure the
terms he uses are appropriately applied in all cases.
36 K A P L A N
SECTION III:
READING COMPREHENSION
LSAT PREP _______________________________________________________________ LSAT Test III Explained: Section III
K A P L A N 37
PASSAGE 1 Asteroids
(Q. 1-7)
Topic and Scope: Asteroids; specifically whether or not its possible for asteroids to have
satellites.
Purpose and Main Idea: The author examines the state of the evidence for asteroid
satellites. While observations of secondary occultations have led some astronomers to
accept the existence of such satellites, others remain unconvinced and are looking for
specific further evidence (well-behaved secondary events).
Paragraph Structure: Paragraph 1 introduces the topic, saying that while most
astronomers used to think it was impossible for asteroids to have satellites, theoreticians
knew all along that such a thing was possible. Paragraph 2 supports the theoreticians;
observations have led many astronomers to believe that asteroids can have satellites,
because when asteroids pass in front of stars, something besides the known asteroid
sometimes blocks out the star as well.
Paragraph 3 describes the most convincing ... report of an asteroid that might have a
satellite. When the asteroid Herculina passed in front of a star, the occultation was
preceded by another occultation, which led astronomers to believe that a satellite orbiting
the asteroid had also passed in front of the star. Paragraph 4 notes that, after the Herculina
event, reports of secondary occultations became respectable, but warns that such reports
have grown so numerous that they cant possibly all be accurate. Paragraph 5 concludes by
saying that even astronomers who remain skeptical would be convinced by a
photoelectric record of a well-behaved secondary occultation of a star, one definitely
caused by a body accompanying an asteroid.
The Big Picture:
A firm grasp of the passage doesnt mean absorbing all of its details (you can look
them up if you need to). Rather, it means figuring out what the authors doing in the
textin this instance, providing evidence that suggests that theoreticians have been
correct all along about asteroid-satellite systems.
Although the authors specific main idea isnt entirely clear until youve read
through the whole passage, topic, scope, and purpose are all revealed by line 15. That
makes this passage an ideal place to start work on this RC section, even if you suffer
from science anxiety. Topic, scope, and purpose, after all, are the three things that
you need to grasp as quickly as possible in order to get the passage under control.
LSAT PREP _______________________________________________________________ LSAT Test III Explained: Section III
38 K A P L A N
The Questions:
1. (E)
This passage is about the existence of asteroid-satellite systems: astronomers used to doubt
their existence, but now recognize that they are theoretically possible, and are looking for
definitive proof. (E) correctly notes that theoreticians were on to the existence of asteroid-
satellite systems before astronomers were. Moreover, (E)s idea that astronomers agree on
what would be conclusive proof echoes the final paragraph.
(A) focuses on a detailthe Heruclina event. But the passage is about asteroid-satellite
systems in general, not just about the Herculina observations.
(B) Au contraire. The first sentence of the passage tells us that astronomers long believed
that stable asteroid-satellite systems weren't possible; furthermore, skepticism about the
existence of such systems has decreased.
(C) Au contraire aussi. The Herculina event supported the theoreticians views about
asteroid-satellite systems. Besides, like (A), this choice is focused on a detail.
(D) is a half-right, half-wrong choice. Skeptical astronomers arent waiting for new
theoretical models, but for physical evidence in the form of photoelectric records.
In global questions, beware of choices that focus on details, contradict the text, or are
only half-right.
2. (D)
Heruclina is discussed in Paragraph 3. It says there that astronomers watching the
Herculina event were surprised by an unexpected drop in brightness that occurred before
the drop they were expecting. Thats (D): evidence of Herculina having a satellite was
provided by the occultation that occurred shortly before the predicted occultation by
Herculina.
(A) First, the secondary bodys presence was strongly indicated, not "directly observed.
Second, whats important is the fact that a second body also eclipsed the star.
(B) is outside the scope. The passage says nothing about a planet near Herculina.
(C) is also outside the scope. The author never discusses the amount of time needed to
complete an orbit.
(E) The occultation of Herculina itself is irrelevant; the evidence for the existence of a
satellite was provided by the secondary occultation.
When working on an explicit text question, always go back to the passage and reread
the relevant portion of text. Never answer on a hunch or vague recall of the text.
LSAT PREP _______________________________________________________________ LSAT Test III Explained: Section III
K A P L A N 39
3. (A)
The attitude of astronomers since the Herculina event is discussed in paragraphs 4 and 5.
Paragraph 4 says the Herculina event made secondary sightings respectablei.e many
astronomers came to accept the possible existence of asteroid-satellite systems. Paragraph 5
indicates that even astronomers who are still doubters would be convinced by the right
kind of evidence. As (A) puts it, astronomers who were skeptical of the existence of
asteroid-satellites have become more open-minded, although many are still awaiting
proof.
(B) describes the attitude of many astronomers prior to the Herculina event.
(C) Theres no chaotic mix of theory relating to asteroid satellites; paragraph 1 indicates
that a single, simple theory supports their existence. Further, the passage doesnt say that
any data is spurious.
(D) is too negative. Many astronomers already believe that asteroid satellites exist. The
skeptics are merely looking for one particular kind of evidence; they arent rejecting all
data not recorded automatically by state-of-the-art instruments.
(E) Theres nothing in the passage about admiration for the scientific process, nor has there
been incontrovertible proof of anything.
When questions ask about attitude, look at the tone of the choices. The words
contempt in (B), bemusement in (C), hardheaded skeptism in (D), and
admiration in (E) all lead to a quick rejection of their choices.
4. (C)
(C) gets to the heart of the matter. The first sentence of paragraph 4 says that after the
Herculina event secondary occultations became respectableand more commonly
reported. This implies that before the Herculina event secondary occultations werent
considered respectable, and so werent commonly reported.
(A) Au contraire. Paragraph 1 indicates that a good theoretical model of asteroid-satellite
systems did exist prior to the Herculina event.
(B) The author never implies that satellite collisions were mistaken for occultations.
Theres no speculation on what, other than an actual satellite, might have occasioned the
rare reported observations of secondary events before the Heruclina event.
(D) Prior to the Herculina event, it wasnt even respectable to report secondary events. The
issue of what constitutes a well-behaved event only arises later (in Paragraph 5).
LSAT PREP _______________________________________________________________ LSAT Test III Explained: Section III
40 K A P L A N
(E) Au contraire aussi. Prior to the Herculina event, it wasnt respectable to report the
observation of a secondary event.
Take note of the time-frame of the question; here youre only interested in the state of
affairs before the Heruclina event.
5. (D)
Paragraph 4 says that reports of secondary occultations grew so common after the
Herculina event that theyre now too numerous for all to be correct. Why? Because even if
every asteroid has the highest plausible number of satellites, only one in every hundred
primary occultations would be accompanied by a secondary event. So (D) must be
correct: Since the Herculina event, reports of secondary events have been occurring at a
rate greater than this maximum plausible rate of one in every one hundred cases.
(A) and (C) simply cant be concluded based on the passages information. We dont know
how many reports of primary occultations have included secondary occultations, or how
many reports of secondary occultations there are or were, so we cant calculate the increase
of either.
(B) distorts the last parenthetical clause of paragraph 4, which describes what would be the
case if asteroid-satellite systems resembled planet-satellite systems, not what is in fact the
case on actual reports of secondary occultations.
(E) is beyond the scope. The passage never mentions any report containing more than one
secondary occultation.
When you come across numbers in the question stem, make sure you understand
the authors purpose in using themin this case, to give you the highest reasonable
limit for the number of secondary occultations one would expect to occur.
6. (C)
(C) is right on the money: the authors primary purpose is to trace the development of
ideas among astronomers concerning the existence of asteroid-satellite systems.
(A) Much of the passage describes how reporting secondary occultations has become
respectable.
(B) deals only with information in paragraph 5. Moreover, (B)s distinction between
spurious and theoretically believable observations isnt made in the passage.
(D) The author isnt trying to bring a theoreticians perspective to the discussion;
instead, hes primarily interested in how experimental results enlighten the disucssion.
(E) is easy to eliminate, because it never even mentions asteroid satellites. Moreover, theres
no attempt to limit speculation about occultation.
LSAT PREP _______________________________________________________________ LSAT Test III Explained: Section III
K A P L A N 41
Quickly eliminate choices that contain terminology that doesnt find an immediate
echo in the passagee.g., the limits of reasonable speculation in (E).
7. (C)
Based on paragraph 5, you can expect the answer to have something to do with the
photoelectric record. (C) would provide the hard physical evidence we need; a
photoelectric record of a well-behaved secondary event is exactly what skeptical
astronomers say would change their minds.
(A) The existence of such early reports is only hinted at, and the clear implication is that
nobody took them very seriously.
(B) The author never implies that theres anything wrong with the original theoretical
model.
(D) is just a more refined abstractionit doesnt constitute the kind of physical proof
needed to resolve the question.
(E) distorts the passages final sentence, which suggests that airplanes passing in front of the
instruments might be responsible for some observations of secondary events.
The airplane vs. ground-based comparison in (E) should have struck you as a totally
new ideadont hesitate to reject such out-of-the-blue choices.
LSAT PREP _______________________________________________________________ LSAT Test III Explained: Section III
42 K A P L A N
PASSAGE 2 17th-Century Scientific Discovery
(Q. 8-15)
Topic and Scope: Scientific experimentation in 17th-century England; specifically, who
performed experiments and why.
Purpose and Main Idea: The author seeks to show how the social prejudices and scientific
views of 17th-century English scientists led them to leave much of their lab work to
technicians, and prevented them from giving technicians the proper credit for their work.
Paragraph Structure: Paragraph 1 states that a distinction must be made between the way
scientific experimentation was described by 17th-century English scientists and the way it
was actually performed. In theory, 17th-century scientists believed that experiments
should be performed by the scientists themselves, without relying on others for assistance.
Yet, as paragraph 2 makes clear, scientists often did not act in accordance with their beliefs.
As the example of Robert Boyle shows, many of them were aided in their experiments by
paid technicians whose contributions went unacknowledged.
Paragraph 3 gives three reasons why the role of technicians was unacknowledged: (1) the
belief that scientific breakthroughs occur as a result of flashes of insight on the part of
brilliant individuals rather than through group efforts; (2) the fact that 17th-century
English scientists were members of the upper class who held the manual labor done by
their technicians in disdain; and (3) the tendency to disregard as unreliable the input of the
wage-slave technicians.
The Big Picture:
Since topic, scope, and purpose all appear in the first sentence, this passage is
another candidate to be worked on early in the section.
Keywords can help you negotiate a passage easily. The long third paragraph is neatly
divided by keywords. The question at the beginning introduces the paragraphs
focus, and the three answers given are clearly marked by the words one reason,
moreover, and finally.
LSAT PREP _______________________________________________________________ LSAT Test III Explained: Section III
K A P L A N 43
The Questions:
8. (C)
(C) covers the gist of the passage. The author explains how much of 17th-century
experimental work was performed by technicians, and how that work was denigrated and
distrusted for the reasons discussed in paragraph 3.
(A) ignores the major concern of the passage: why technicians contributions were
overlooked. Moreover, the passage never implies that scientific experiments would have
been absolutely impossible without the aid of technicians.
(B) distorts the passage. Lab workers, as salaried employees, belonged to a lower social
caste.
(D) is outside the scope. The passage never discusses the relationship between 20th-century
scientists and their technicians.
(E) focuses on a detail.
In global questions, its not enough to look for a choice that sounds like it came from
the author. Youve got to find the choice that encompasses the topic, scope, and
purpose of the text.
9. (E)
Seventeenth-century rhetoric about scientific experimentation emphasized the idea that
scientists should do their own experiments. Hence, that rhetoric would have more
accurately described the work conducted in Boyles lab if Boyle himself had actually done
his own experiments.
(A) would contradict seventeenth-century rhetoric by having Boyle admit that he relied on
others.
(B) According to the passage, contempt for manual labor was characteristic of seventeenth-
century scientists.
(C) Membership in the Royal Society alone wouldnt determine whether or not Boyle
performed experiments according to the rhetoric of the day.
(D) focuses on the wrong issue. The rhetoric in question was about scientists performing all
their own experiments, not about acknowledging technicians; acknowledging technicians
is an issue raised by the author.
The main difficulty here is in sorting out the question. Take it one step at a time:
First, what is the rhetoric the question speaks about? Second, what does it want you
to do with the rhetoric? (It wants you to find a situation in Boyles lab that fits the
rhetoric.)
LSAT PREP _______________________________________________________________ LSAT Test III Explained: Section III
44 K A P L A N
10. (C)
The franchise and its relation to servants is discussed in the middle of paragraph 3,
where its said that servants were excluded from the franchise because, as wage earners,
they were thought to be controlled by their employers. In other words, as (C) says, their
political independence was thought to be compromised.
(A) Servants werent excluded because their interests were already represented, but
because it was believed that they would blindly support the political positions of the
employers who paid their wages.
(B), (D), and (E) are outside the scope. Nowhere does the passage state or imply that
servants were inadequately educated (B), a polarizing force (D), or insufficient
contributors to society (E).
Dont worry about the precise definition of the franchise; its enough to locate the
word in the passage and see how it appears in context.
Even if you didnt remember where the concept of franchise appeared, it should
have been easy to find, given the paragraph divisions of the passage; paragraph 3 is
where the author gets around to discussing the social reasons technicians were
looked down on.
11. (D)
In the middle of Paragraph 1, were told that the Royal Society of London endorsed the
notion that doing menial work in the cause of science was a good thing. As (D) puts it, the
Society advocated abandoning the traditional upper class ethic against performing manual
laborat least as far as science was concerned.
(A) and (E) are au contraire choices. Pararagraph 1 explicitly says that, as far as the Royal
Society of London was concerned, the willigness of scientists to do their own manual labor
was part of an attempt to discover Gods truth in nature (A), as well as a demonstration of
piety (E).
(B) and (C) touch on issues which the Royal Society didnt address. The Society never
asked scientists to abandon the individualistic view of scientific breakthroughs (B) or the
view that wage-dependent servants shouldnt vote (C). In fact, based on the information in
the passage, its quite likely that the upper class Royal Society would have endorsed these
views.
If you didnt reread the relevant portion of text, you might easily have fallen for one
of the trap choices.
LSAT PREP _______________________________________________________________ LSAT Test III Explained: Section III
K A P L A N 45
12. (A)
The second sentence of paragraph 3 indicates that both the seventeenth and twentieth
centuries share the view that scientific discoveries result from the sudden insights of a
small number of brilliant individuals, rather than from the cooperative efforts of many
people.
(B) is outside the scope. Theres nothing in the passage about either seventeenth or
twentieth century views of the connection between political values and scientific method.
(C) and (D) are out because, while the beliefs that research undertaken for pay couldnt be
objective (C) and that scientific discovery could reveal divine truth (D) were indeed both
held in the seventeenth century, the passage says nothing about twentieth-century beliefs
concerning these things.
(E) states a position that certainly wasnt endorsed in the seventeenth century (and may not
be widely endorsed in the twentieth century, for all we know).
In inference questions, beware of choiceslike (B) herethat are outside the
authors scope.
13. (D)
Paragraph 3 begins by asking why the role of technicians wasnt acknowledged by 17th-
century scientists, an issue introduced in Paragraph 2. It then answers this question by
discussing three factors that contributed to the failure to acknowledge the role of
technicians.
(A) The question isnt posed in the previous paragraph, nor is only one of several
alternative answers adopted.
(B) None of the factors discussed in paragraph 3 is rejected.
(C) The explanations discussed arent incompatiblethe factors in paragraph 3 are shown
as working together.
(E) Paragraph 3s explanation doesnt rest on recent research, but is based on the authors
interpretation of seventeenth-century English society.
Reject choices as soon as they deviate from what you know about the passagee.g.,
reject (A) as soon as you see question posed in the previous paragraph and (E) as
soon as you see recent discoveries.
LSAT PREP _______________________________________________________________ LSAT Test III Explained: Section III
46 K A P L A N
14. (A)
In introducing the political significance of the wage relationship, the author makes the
point that workers dependent upon the wages of their employers simply werent
considered reliable, whether in political judgments or in scientific research. As (A) says,
the author puts the scientists failure to acknowledge the contributions of technicians in the
context of general worker-employer relations.
(B) is outside the scope. The author offers no general thesis about the relationship between
scientific discovery and economic conditions.
(C) goes against the gist of the passage, which is that seventeenth-century scientists relied
on technicians to do much more than simply the most menial tasks.
(D) is also outside the scope. The author doesnt discuss political or economic changes in
seventeenth-century England.
(E) The author doesnt tie the wage relationship to the nature of scientific discovery, but to
the attitude of scientists towards technicians.
Always read around the line cited to get the context. Here, you get the answer
virtually handed to you in lines 53-55.
15. (D)
As weve already seen, the rhetoric of seventeenth-century English science concerned the
idea of doing hands-on research. What was the rhetoric? That scientists should conduct,
observe, and analyze their own experiments.
(A) The myth about how discoveries were thought to occur is a general trend discussed by
the author, not a subject of seventeenth-century rhetoric.
(B) Au contraire. Seventeenth-century scientific rhetoric emphasized the importance of
doing manual labor in the cause of science.
(C) Seventeenth-century scientists failed to acknowledge the contributions of their
technicians. The scientific rhetoric of the time never addressed that issue.
(E) Though scientists like Boyle did believe in the search for divine truth in nature, the
rhetoric discussed in the passage concerned only the importance of manual labor in
research.
Eliminate odd-man out choices quickly. In this case, the rhetoric in question clearly
has to do with the manner in which scientific research takes place, so throw out (E)
right away.
LSAT PREP _______________________________________________________________ LSAT Test III Explained: Section III
K A P L A N 47
PASSAGE 3 Monopoly Power
(Q. 16-20)
Topic and Scope: Monopoly power; specifically, the difference between the possession of
monopoly power, which isnt illegal, and the abuse of monopoly power, which is.
Purpose and Main Idea: The authors purpose is to describe what sort of exercises of
monopoly power are considered violations of federal antitrust laws. Since this is a
descriptive passage, there really isnt a specific main idea.
Paragraph Structure: Paragraph 1 explains that the possession of monopoly power is not
in itself illegal; to violate antitrust laws, a company must abuse monopoly power by using
it to exclude competition.
Paragraph 2 explains how monopoly power comes about: Companies with a large market
share can raise prices above competitive levels without losing customers. Paragraph 3
explains why the mere possession of monopoly power isnt illegal: Tighter laws might pose
disincentives to the growth of monopolies and impair consumers welfare. Paragraph 4
describes the types of exclusionary practices which constitute abuse.
Paragraph 5 reiterates a point made earlierthat, in the interests of consumer welfare,
antitrust laws focus on the abuse rather than the possession of monopoly power.
The Big Picture:
Be on the lookout for passages that contrast two or more entities: the possession vs.
the abuse of monopoly power, for instance. Such passages always have questions
that hinge on a clear understanding of the difference between the entities being
compared.
You dont have to assimilate all of the details to do well on this passage. The
important thing is to understand the basics of monopoly powerwhats illegal
(abuse), whats not (possession) and why (consumer welfare).
LSAT PREP _______________________________________________________________ LSAT Test III Explained: Section III
48 K A P L A N
The Questions:
16. (E)
Paragraph 5 says that antitrust laws focus on abuse of monopoly power rather than
possession of it in order to protect consumers welfare. So we can infer (E): that the abuse of
monopoly power is prohibited because it impairs consumer welfare, whereas possession
doesnt necessarily hurt consumers.
(A) The author doesnt make a legal distinction between market share and market control.
(B) The author never suggests that monopoly power is easier to demonstrate than abuse.
(C) In the first paragraph, abuse of monopoly power is defined as the exclusion of
competition in the monopolized market or related markets, so it neednt involve more
than one market.
(D) Paragraph 3 says that charging supracompetitive prices doesnt by itself constitute an
abuse of monopoly power.
Watch out for categorical choices like (C), which says abuse of monopoly power
must involve more than one market. Compare this choice with correct choice (E),
which says that monopoly power doesnt necessarily hurt consumer welfare.
17. (E)
At the beginning of paragraph 4, leverage is described as the use of power in one market
to reduce competition in another, a strategy which is clearly characterized as abuse in
paragraphs 1 and 4.
(A) Au contraire. Were told that the manipulation of related markets constitutes abuse,
even though these secondary markets arent monopolized.
(B) Tying arrangements are presented as an example of leverage strategy, and all
leverage strategies are considered abuses by the author.
(C) Au contraire aussi. The use of monopoly power in itself doesnt constitute abuse.
(D) A company using leverage would still violate antitrust laws, even if it was charging
competitive prices.
Once you see that leveraging is basically defined as an abuse of monopoly power,
you can eliminate the two nos, (A) and (B), and the qualified yes, (D).
18. (D)
The passage as a whole revolves around the distinction between possession of monopoly
power and its abuse. In the third paragraph, the author brings up a number of cases where
companies that possess monopoly power use it legally. The author is clarifying how far
LSAT PREP _______________________________________________________________ LSAT Test III Explained: Section III
K A P L A N 49
companies can legally exercise monopoly power. As (D) puts it, the author is
distinguishing what is covered by the antitrust laws from what isnt.
(A) is outside the scope. Theres no mention of supracompetitive profits in the passage, if
indeed such profits exist.
(B) distorts the passage. Were told how far companies can exercise monopoly power
without breaking the law, a quite different thing from describing positive uses of monopoly
power.
(C) focuses on a detail, not the main purpose of the paragraph.
(E) doesnt really emerge until paragraph 5; its by no means the central idea of paragraph
3.
This question is a natural for pre-phrasing, one of the most useful question-
answering techniques. In this case, your grasp of paragraph structure should have
clued you in that the answer would focus on illustrating when the use of monopoly
power isnt illegal, which leads right to (D).
19. (B)
Essentially the lawmakers attitude toward monopoly is that some methods of reducing
competition are legitimate, and some arent.The point of the passage, after all, is to
distinguish between legal and illegal forms of monopoly. Paragraphs 2 and 3 center
around the extent to which companies can exercise monopoly power without violating
antitrust laws. Paragraphs 4 and 5, on the other hand, focus on uses of monopoly power
that are prohibited by antitrust laws.
(A) Au contraire. At the end of paragraph 3, were told that monopolist companies can be
allowed to grow at the expense of competition in the interests of consumers welfare.
(C) The author says that consumer welfare is the principle aim of the antitrust laws.
(D) Au contraire aussi. According to paragraph 2, when close substitutes for a product are
available, competition benefits from a company that charges supracompetitive prices.
(E) Since the existence of monopolies is considered better for consumer welfare under
certain circumstances, lawmakers presumably wouldnt agree that competition is
necessary to supply high-quality products at low prices.
Use what you learn in dealing with one answer choice to help you evaluate the
others. Here, once you see that its a mistake to tie competition to consumers welfare,
its easy to reject the wrong choices.
LSAT PREP _______________________________________________________________ LSAT Test III Explained: Section III
50 K A P L A N
20. (A)
The authors point in the final paragraph is that the legal distinction between possession of
monopoly power and its abuse is based on a desire to promote consumer welfare. We want
a choice thats relevant to this idea. (A) fits the bill by picking up the consumer-welfare-
based distinction between abuse and possession of monopoly power, and explaining that
monopoly power can sometimes be in the consumers best interests.
(B) goes against the gist of the last paragraph. The author believes that antitrust laws have
been effective in securing the consumers best interests.
(C) is outside the scope. It focuses on two particular industries that havent been
mentioned anywhere.
(D) dredges up the idea of supracompetitive profits, which arent mentioned anywhere
in the passage.
(E) also goes against the gist of the paragraph. Restraints on monopoly havent been left to
the market, but rather have been enforced by antitrust laws.
Finding the sentence that logically completes a paragraph is not difficult if you keep
the paragraphs purpose in mind.
LSAT PREP _______________________________________________________________ LSAT Test III Explained: Section III
K A P L A N 51
PASSAGE 4 Navaho Blankets
(Q.21-28)
Topic and Scope: Navajo weaving; specifically, the different styles of Navaho rug weaving
and how they developed.
Purpose and Main Idea: The author describes a theoryAmsdens theory about Navajo
weaving styles and how they evolvedand then calls that theory into question.
Paragraph Structure: Paragraph 1 introduces Amsdens view of Navajo weaving styles:
three of them are banded with stripes, zigzags, or diamonds, while the fourth style is quite
different, a border surrounding central figures. Paragraph 2 explains that Amsden believes
that theres some Anglo influence in the diamond style, but the most Anglo influence
appears in the bordered style. Paragraph 3 gives the meat of Amsdens argument: he
believes that the bordered rug represents a radical break with previous styles, and that the
very fact of the border changed the way Navajo weavers designed rugs.
Paragraph 4 begins the authors criticsm of Amdens theory: Amsdens view raises several
questions. First question: what is involved in altering artistic styles? The author concludes
that in the case of weaving, theres no radical change in motor habits or thought processes.
Paragraph 5 raises the second question: whats the relationship between banded and
bordered styles? The author contends that the break in style isnt a break in psychology,
but a result of the artists quest for invention. Finally, Paragraph 6 questions the idea that
there really is a stylistic gap between banded and bordered styles.
The Big Picture:
When more than one view is presented, you need to be clear about the distinctions
between or among the different points of view. The questions will certainly test to
see that youve grasped the differences.
Notice how neatly this passage is arranged. The first three paragraphs describe
Amsdens views, while the last threebeginning at line 28supply the authors
critique of Amsden.
LSAT PREP _______________________________________________________________ LSAT Test III Explained: Section III
52 K A P L A N
The Questions:
21. (E)
This choice captures the passages focus on Amsdens views of Navajo weaving styles and
the authors critique of those views.
(A) is outside the scope. The author never suggests that the Navajo rejected all Anglo
cultural influence.
(B) distorts the authors criticism. The author questions Amsdens account of how the styles
developed, but doesnt reject Amsdens categorization of the styles.
(C) focuses on a detail. It plays on an idea that the author puts forth in paragraph 5.
(D) is outside the scope as well. It doesnt even mention Navajo weaving, Amsden, or the
author.
Avoid choices that are either too broad or too narrow in scope.
22. (A)
The author mentions the strips of color breaking through the enclosed border as evidence
of Navajo distaste for the Anglo preference that graphic designs have a top, bottom, and
border. (A) paraphrases this sentiment, albeit in abstract language.
(B) Amsden depicts the strips of color as signs of general Navaho abhorrence for borders,
not necessarily an echo of the diamond style.
(C) Au contraire. For Amsden, the strips of color bursting through the border reflect
resistance to Anglo culture.
(D) According to Amsden, the Navajo resisted the bordered style, not the banded style.
(E) The desire for designs with a top, bottom, and border is presented as an Anglo desire.
Be suspicious of any choice that has a highly charged word like disintegrate (D),
unless the tone of the passage clearly warrants such a word.
23. (C)
(C) captures the authors view as it is expressed in paragraphs 5 and 6. The author thinks
that the bordered style gradually evolved from the banded style not necessarily as a result
of Anglo influence, as Amsden believed, but as a result of Navajo experimentation with
design and the artistic quest for invention.
(A) In mentioning the Chief White Antelope blanket at the end of the passage, the author
suggests that the diamond style pre-dated the arrival of the Anglos.
LSAT PREP _______________________________________________________________ LSAT Test III Explained: Section III
K A P L A N 53
(B) is outside the scope. Theres no evidence of what the author thinks is generally the
case when two cultures occupy the same region.
(D) is also outside the scope. Nothing in this passage suggests that non-Anglo cultures
influenced Navajo weaving.
(E) Saying that vertical arrangements of diamond parts anticipated the border isnt the
same as saying that rows of horizontal and vertical diamonds were transformed into
solid lines to create the border.
Dont spend an inordinate amount of time with any choice that confuses youas (B)
may have done here. Move on quickly to evaluating the other choices.
24. (D)
What happened in 1890? The bordered style appeared. So, youre looking for the choice
thats not characteristic of pre-1890 weavingsi.e., thats uniquely characteristic of the
bordered style. The bordered style used isolated figures (paragraph 1), while pre-1890
weavings used continuous patterns (paragraph 3).
(A) Paragraph 3 says that the old patterns alternated decorations like stripes, zigzags, or
diamonds in a regular orderthats a repetition of forms.
(B) Early Navajo rugs were continuous, with overall patterns rather than isolated figures.
(C) and (E) According to paragraph 1, horizontal bands and color were used in early
styles.
In all/EXCEPT questions, youre looking for the choice that isnt true.
25. (D)
The author accepts Amsdens classification that Navajo weavings used horizontal bands of
abstract designs early on, and later moved on to isolated figures (when the bordered style
was adopted); but differs from Amsden in arguing that the change was gradual, not a
radical break.
(A) In Paragraph 4, the author suggests that motor habits and thought processes have little
application to Navajo weaving.
(B) Neither the author nor Amsden attributes the zigzag style to Anglo influences.
(C) Au contraire. The figures came later.
(E) Also au contraire. Only the border style is said routinely to contain isolated figures.
In an open-ended question like this one, its difficult to prephrase. The only course
to take is to go through the choices one-by-one.
LSAT PREP _______________________________________________________________ LSAT Test III Explained: Section III
54 K A P L A N
26. (C)
The authors main point of issue with Amsden concerns the claim about Anglo influence.
The authors objections occur in paragraphs 4, 5, and 6. The authors basic point here is that
Amsden has overlooked some thingsthe nature of weaving, the artists quest for
invention, the existence of intermediate formsthat suggest that the bordered style may
have arisen without Anglo evidence. As (C) says, the author thinks that Amsden fails to
consider certain aspects of Navajo weaving in making his claim.
(A) is clearly incorrect because Amsden sees little or no correspondence between Anglo
and Navajo art. Rather, he views the two styles as radically different.
(B) is outside the scope. The author never discusses Amsdens feeling about Anglo culture.
(D) The author doesnt criticize Amsden for basing his theories on a limited number of
weaving specimens.
(E) is also outside the scope. There is no suggestion that Amsden has confused the features
of the zigzag and diamond styles.
In inference questions, dont endorse choices that stray too far from the passages
content.
27. (B)
In the final paragraph, the author makes the point that some stylistic changes that led the
way to the border style can't be attributed to Anglo influence, and uses the Chief White
Buffalo blanket as an example to illustrate this point.
(A) Au contraire. The Chief White Antelope blanket argues against the influence of Anglo
culture on the bordered style.
(C) Au contraire, too. The author says that the vertically arranged diamonds in the Chief
White Antelope blanket anticipate the border. Moreover, this blanket has a flowing
design, not a central design.
(D) The Chief White Antelope blanket questions the idea of Anglo influence.
(E) This blanket seems to illustrate innovation within the diamond style.
In questions that ask for the why of a detail, the answer is to be found by reading
the lines around the detail. Be sure to get a sense of the details context before
shopping among the answer choices.
LSAT PREP _______________________________________________________________ LSAT Test III Explained: Section III
K A P L A N 55
28. (B)
Paragraphs 1 through 3 describe Amsdens view about how the bordered Navajo weaving
style developed. Paragraphs 4 through 6 question that view. Thus, this choice accurately
sums up the passages primary concern.
(A) Although the passage does compare different weaving stylesthe banded and the
borderedthe central concern is not to compare the styles, but to question a view
regarding the development of these styles.
(C) is outside the scope. The passage never proposes new methods of investigation.
(D) This choice focuses on a detail. While the author does discuss the influence of Anglo
style on Navajo weaving, thats done only in order to question Amsdens explanation of how
the bordered style evolved.
(E) The authors focus is on the evolution of a style. The interaction between two cultures
is an idea of Amsdens.
In questions that ask for the authors main concern or primary purpose, start with a
verb scan to eliminate wrong choices. In this case, its important to realize that the
author isnt neutral; he or she is disagreeing with a view. So, neutral verbs like
comparing, discussing, and analyzing wont do.
56 K A P L A N
SECTION IV:
LOGICAL REASONING
LSAT PREP _______________________________________________________________ LSAT Test III Explained: Section IV
K A P L A N 57
1. (D)
The only way for a violin to play recognizably the same music as the piano is for the
violinist to keep the nature and possibilities of his instrument in mind, and to try to suit
the original composition to the new instrument. Since the two different instruments are
intended to be analogous to the two different languages, the point of the argument must be
that translation from one language to another can only be achieved if the translator does
not simply try to copy the original exactly, but is also guided by the innate possibilities
and limitations of the new language.
(A) No; by the very nature of the analogy, the author is implying that poetry tranlation is
possible, so long as the translation is guided by the nature of the language its translated
into.
(B) attempts to confuse the issue by combining the two elements of the analogy, music and
poetry, byt the argument is not even remotely concerned whether some languages are
more or less musical, and hence more or less poetic, than others. It also strays from the
scope, translation.
(C) also tries to combine poetry with musical qualities like rhythm and sound patterns,
therefore confusing the purpose of the analogy, which isnt included to convince us that
poetry should be like music, but rather to suggest that similar factors that influence
musical translation may also influence poetry translation.
(E) is plainly outside the scope of the argument, which never debates the relative difficulty
of translating philosophical insights or subjective impressions.
Understand that the purpose of an analogy is to show or imply one element of
similarity between two different things, not to argue that the two things are or
should be similar in every respect.
Read critically and always keep in mind what the author considers important. Ask
yourself, what would the author like me to take away from this passage? In a
question like this, any choices that dont include the concept of translation, the main
scope of the argument, should be quickly discarded.
2. (C)
According to the author, if a students understanding of a subject consisted only of
knowing facts and rules, then computers might well eventually replace human teachers as
drill masters and coaches. However, a student's understanding also consists of having a
grasp of the general concepts underlying them. The author concludes that the computer
will not eventually replace the teacher, obviously assuming that computers cant teach
these general concepts along with the facts and rules. (C) undermines the argument by
attacking this assumption.
(A) asserts that computers are as good as teachers in drilling students on rules and facts,
and (D) goes one step further and says that theyre better. The author freely admits that
computers would be capable of replacing teachers if thats all there is to it, but argues that
computers fall short because learning requires a grasp of underlying concepts as well.
LSAT PREP _______________________________________________________________ LSAT Test III Explained: Section IV
58 K A P L A N
(B) Au-contraire; this actually supports the argument by stating that the teacher's essential
task is to make students understand the general concepts behind specific facts and rules
exactly what the author claims computers are not able to do.
(E), if anything, supports the argument, because the author implies that drills and coaching
are the computers strong point.
One of the most common ways to weaken an argument on the LSAT is to call the
authors central assumption into question.
3. (C)
Heres the reasoning: If city council spending (X) remains the same, then the sales tax (Y)
can be expected to be at a certain level. Therefore, if the sales tax (Y) is higher than the
predicted level, then it will be because city council spending (X) increased. This element of
causation throws a kink into the logic. The contrapositive of the first sentence is If the sales
tax is not 2%, then city council didnt maintain spending at the same level. The higher tax
mentioned in the second sentence could be the result of other factors; it need not be caused
by an increase in expenditures. (C) most closely parallels this, complete with the logical
flaw: If workers wages (X) arent increased, then the prices charged for goods (Y) will
remain at the level it was at last year. Therefore, if the prices (Y) rise beyond the predicted
level, then it will be because the workers wages (X) also increased.
(A) begins with a premise relating the cost of house-building to the price of houses, but
goes on to try to deduce something about builders attempting to sell a greater number of
houses, which is a completely new idea.
(B) also complicates things by introducing the idea of reduced profits due to shoplifting, a
third term in the argument.
(D) Much like in (B), (D)s conclusion bears no resemblance to the stimulus conclusion,
since it introduces the totally new idea of improved services, a third term not found in the
original.
(E) is concerned with qualitative terms, whereas the stimulus was concerned with
quantitative terms. (E) also drags a new consideration, newspaper circulation, into its
conclusion.
Its often quite valuable to do a quick scan and comparison of the arguments and
choices conclusions in Parallel Reasoning questions. If the originals conclusion is
that if something happens it will be because something else has happened,
chances are that the correct choice will follow this it will be because format. (A)
and (C) are the only viable candidates to consider.
Assess the merit of the argumentif the original seems to botch a cause-and-effect
relationship, then look for the choice that does the same.
LSAT PREP _______________________________________________________________ LSAT Test III Explained: Section IV
K A P L A N 59
If you can symbolize the argument algebraically, do so, and pay careful attention to
the number of terms represented in the original; many of the wrong choices will
include additional or fewer terms.
4. (D)
Good deeds are beneficial to the immune system of the person who does them. White
blood cells are needed to fight infection, and magnanimous behavior causes the brain to
produce chemicals that stimulate and aid the activity of these white blood cells. We are
therefore safe in inferring that magnanimous behavior is beneficial to ones own interests.
(A) The stimulus says nothing about what constitutes a good deed, or what sort of motives
are required for a deed to be truly good.
(B) While its claimed that magnanimous behavior helps the immune system, we can
hardly say that lack of magnanimity actually causes most serious illnesses.
(C) is way off. Magnanimity produces the chemicals; not the other way around.
(E)s a distortionthe stimulus said only that magnanimity will stimulate the activity of
white blood cells, so we cant infer that it will produce new ones.
Beware of choices with extreme or radical language that doesnt fit the tone of the
stimulus. Distortions, or extreme sounding choices are some of the most common
wrong answers.
Focus on the path of causality when one thing is said to cause another. Very often,
wrong choices can be eliminated because they switch the cause-and-effect
relationship. Magnanimity causes the stimulation of beneficial chemicals related to
the immune systemwrong choice (C) gets it totally backwards, while (B)
misunderstands the relationship as well.
5. (C)
The author concludes that if production costs for operas were lowered, then operas would
no longer need corporate sponsorship, but instead could be privately financed; as a result,
ticket buyers would be able to see a wider variety of operas, instead of just the most
famous ones. (C) destroys this wishful thinking. If, without corporate support, opera
companies could still afford to produce only the most famous operas, the argument falls
apart.
(A) subtly strengthens the argument, by opening up the possibility that people would be
willing to pay to see little known operas as opposed to famous ones.
(B) The argument isnt affected in the least if corporate sponsors still wish to support
opera, the whole issue centers on what will happen without corporate sponsorship.
(D) The authors whole point is to produce the operas without corporate sponsorship, so
this piece of info has no bearing on the argument.
LSAT PREP _______________________________________________________________ LSAT Test III Explained: Section IV
60 K A P L A N
(E) We can infer that the author has already thought of this, which is why her plan involves
cutting production costs in order to produce operas without corporate support.
A simple restatement of something that the author is taking for granted as true (i.e.
an assumption) doesnt strengthen or weaken an argument; only a choice that breaks
down or shores up this assumption can affect the argument in this way.
Dont underestimate the power of asking yourself so what? when evaluating
choices in strengthen/weaken questions; many choices in this question type are
simply irrelevant, and therefore have no effect on the authors logic.
6. (A)
The sentence at the same time that the clock opened up some avenues, it closed others
gives it away; it introduces the notion of something having both a liberating and a
restricting effect. Since were looking for a proposition (a generality) illustrated by this
example, (A) fits the bill by expanding the authors observation about the double effect of
clocks and applying it to new machines in general.
(B) is never suggested, and besides, were looking for a general concept that can be
illustrated by the clock example, not a recommendation of what to do.
(C) The clock example doesnt match this at all, as it illustrates that new technologies can
improve our lives, even those that also have a restricting effect.
(D) The author does not offer a final judgment about clocks, and once again, since were
looking for a generally-stated proposition based on the clocks example, any choice
containing only a clocks-related soundbite must be wrong.
(E) mistakenly takes a specific attribute of clocks, the fact that they increase
synchronization and productivity, and attributes it to most machines. The actual
functioning of the clock, and the specific effects it has (like adding synchronization),
cannot be generalized and applied to other machines, which may, after all, have entirely
different functions and effects.
Always keep the scope of the argument in mind. Many (if not most) of the wrong
choices will simply go beyond (or way beyond) what the author is saying.
When you see the words proposition or principle, think more in terms of
generalities than specifics.
LSAT PREP _______________________________________________________________ LSAT Test III Explained: Section IV
K A P L A N 61
7. (B)
The author of the argument has erroneously combined the premises in the first two
sentences and taken them to mean that a person must practice three hours a day to become
an expert. This is a case of confusing necessity with sufficiency. True, according to the
second sentence, three hours a day of practice will make someone an expert eventually
(i.e., its sufficient for expertise), but nothing rules out the possibility of becoming an
expert by practicing less than three hours a day (i.e., three hours a day is not necessary).
According to the first sentence, at least some practice is necessary, but just because three
hours a day will ensure expertise doesnt mean that someone couldnt acquire expert status
with only one hour of practice a day.
(A) Were told that a person who practices three hours a day will eventually become an
expertit therefore doesnt matter if someone who practices three hours a day is not yet
considered an expert.
(C) is the contrapositive of the conclusion, and therefore makes the same error that the
conclusion makes, that of assuming that one must practice three hours a day to become an
expert.
(D) The argument doesnt specify that the practice hours must be consecutive, so the fact
that some music teachers may frown on this notion is irrelevant.
(E) also brings up an irrelevant considerationthe argument focuses the practice
requirements that will lead to expertise. How many or how few people have this kind of
time on their hands is outside the scope.
Pay attention to, and never lose sight of, what youre asked for. The answer to a flaw
question must contain a flaw. Statements about the argument that are true or simply
restate information must be eliminated.
Dont fault an author for not addressing a point that he has no logical obligation to
address. If the argument proceeds X will lead to Y, the author has no obligation to
also prove that lots of people can do X. Its beyond the scope.
8. (E)
First, define the unexpected finding. Theres incontestable proof that safety seats will
reduce the number of serious injuries sustained by children in car accidents, yet a large
number of children who are riding in safety seats continue to receive the serious injuries
that the seats were specifically designed to prevent, and have been proven to be effective in
preventing. If, as (E) says, safety seats must be used properly in order for them to afford
protection, and if parents are failing to use them properly, then its easily conceivable for
children riding in safety seats to still suffer serious injuries.
(A) and (C) are wrong for the same reason: Both supposed explanations ignore the fact that
the unexpected finding centers around children who are actually in the seats receiving
injuries.
LSAT PREP _______________________________________________________________ LSAT Test III Explained: Section IV
62 K A P L A N
(B) More children making automobile trips doesn't bring us any closer to understanding
the mysterious failure of the seats to perform the function for which they were designed,
and for which they have been proven effective.
(D) This one also explains nothingit ignores the fact that children are sustaining the very
injuries that the seats were designed to prevent.
Not all Paradox questions make use of the word paradox; learn to recognize the
various ways this question type is presented. Unexpected finding, discrepancy,
and surprising result are three common phrases that signify Paradox.
The answer to a Paradox question cannot contradict the stimulus or bring up
irrelevant information (like parents who dont even buy the seats).
9. (A)
Since both the willingness to make fun of oneself and the willingness to allow others to do
so are evidence of a self-confident person, we can infer that people who aren't self-
confident aren't likely to enjoy being made fun of, either by themselves or by others. (A)
implies that no one who lacks self-confidence will enjoy being made fun of, which may at
first seem like a stronger statement than the argument can definitely support. However, the
phrase the surest mark is meant to indicate that telling funny stories or jokes about
oneself is tantamount to being self- confident; in other words, if you tell funny stories or
jokes about yourself, then you are self-confident. (A) is the contrapositive of this. Even if
you dont see eye to eye with this logic, as some may not, (A) is still the best of the bunch,
and very much in line with the gist of the argument.
(B) Beyond the scopethe passage talks about other people making fun of self-confident
people, but says nothing about the reverse scenario.
(C) and (E) are also beyond the scope. Theres no way for us to infer the reason behind the
stated correlation (C); that is, why self-confident people put themselves down. And the
reason why anyone tells jokes about other people in their presence (what choice (E)
discusses) is never discussed.
(D) We have no idea who most people are, or what they would prefer.
In Inference questions, pay attention not only to what you know, but also to what
you dont know. Things that you dont know will appear as wrong answer choices
for sure.
LSAT PREP _______________________________________________________________ LSAT Test III Explained: Section IV
K A P L A N 63
10. (E)
This ones fairly complex, but your best bet is to follow the cause-and-effect relationships
and make deductions as if they represented Logic Games rules. Only one of the choices
cant be deuced from the rules of this stimulus, and it turns out to be (E): We know that if
there's a decrease in atmospheric carbon, there's a decrease in atmospheric heat. We cannot
infer the opposite: If there's a decrease in atmospheric heat, there may or may not be a
decrease in atmospheric carbon.
(A), (D) Straight from the passage: A decrease in atmospheric carbon leads to a decrease in
atmosphere heat, which results in decreased evaporation of sea water. The decreased
evaporation of sea water results in less rain, which means less carbon being washed into the
seas.
(B) If the amount of carbon in the atmosphere increases, then the atmosphere holds more
heat. This increased heat in turn results in increased evaporation of sea water. Put 'em
together, and we get (B): increased carbon in the atmosphere means increased evaporation
of sea water.
(C) More of the same process: Increased heat leads to increased evaporation of sea water,
which leads to increased rainfall.
An inferred EXCEPT question is by nature complex; four out of the five choices
must be inferable from the stimulus, which means that theres a lot of reading
between the lines to be done. One option is to skip this question type (and assumed
EXCEPT questions as well) and come back to it at the end, especially when the
stimulus is based on a long and confusing process like this one.
Dont be afraid to temporarily skip intimidating questions. In order to succeed on
the LSAT, you must dictate to the test, not the other way around.
11. (E)
The author concludes that environmentalists are wrong to worry about increased carbon
levels due to the burning of fossil fuels, because nature will continually adjust the carbon
level. However, if, as choice (E) states, the adjustment process works very slowly, allowing
wide short-term fluctuations in the carbon level, then its conceivable that dangerous or
even lethal levels of carbon can build up, thus severely weakening the author's argument:
the environmentalists would be right to worry.
(A), (C) The author doesnt debate the necessity of carbon, she merely argues that nature
takes care of the carbon level by itself, and that the environmentalists should chill.
(B) Some may feel that this weakens the authors concession in the last sentence, but even if
it did, that sentence is basically a tangential point that has little to do with the logic of the
argument, which deals with natures regulation of the carbon level. However, this probably
doesnt even affect the authors concession, because that statement never said or implied
that the threat to humans of increased carbon levels would come from breathing it in; it
would likely be more indirect, along the lines of the processes described.
LSAT PREP _______________________________________________________________ LSAT Test III Explained: Section IV
64 K A P L A N
(D) This is an irrelevant comparison that has no bearing on the logic of the argument.
Recognize filler material (like the authors concession that a sustained increase in the
carbon level would threaten human life) that plays no real part in the overall logic of
the argument. Theres usually a wrong choice that keys off of such irrelevant info.
12. (E)
Based on the statistical evidence provided, the author concludes that if American children
are to become as capable as their South Korean peers, they must watch less television. The
author doesn't consider the possibility that there are other reasons for this difference in
mathematical ability. Surely how much television children watch isn't the only difference
between lifestyles of South Korean and American children. In making this claim, therefore,
the author assumes that other possible factors that could account for the difference in math
abilities dont in fact play a role. (E) zeros in on this assumption: it certainly would seem
that mathematical instruction would also be an important factor in determining
competence in math; therefore, in concluding that less TV watching will help improve U.S.
childrens math abilities, the author must assume that math instruction in America and
South Korea is of the same caliber.
(A) and (B), if true, both weaken the argument by providing (and not discounting, as (E)
does) other explanations: lack of interest and discipline on the part of American children
compared with the South Koreans. If these things are true, then the impact of TV watching
on the situation may be at best insignificant and at worst irrelevant.
(C) The authors conclusion, simply put, is based on a necessary condition that underlies
U.S. childrens math success. The argument ends there; it stands regardless of whether
children will be motivated to adopt this condition.
(D) The conclusion is stated in terms of watching less TV; how much less is not
addressed. This one hour time frame appears out of nowhere, and the author need not
assume this for the conclusion to stand. Remember, watching less TV is a necessary
condition, according to the author, but is not sufficient for math success, so the author
need not assume that watching less than one hour of TV a day will guarantee an increase in
math ability.
Assumptions often spring from something that the author has failed to consider,
and therefore has assumed to be unimportant. In other words, a very common
assumption on the logical reasoning section is the assumption that another viable
explanation, which could in fact explain the scenario and weaken the authors
contention, is not so. Notice in this case that many of the choices contained
alternative explanations for the discrepancy noted, but only correct choice (E)
negated the possibility of the alternative it provided.
If theres an alternative explanation for the facts of a situation, the testmakers
can write a number of different questions that in some way play off of this other
explanation: logical flaw, strengthen/weaken, or as in this case, assumption. Review, if
necessary, the concept of the synergy between these types of Logical Reasoning
questions, as discsussed in class.
LSAT PREP _______________________________________________________________ LSAT Test III Explained: Section IV
K A P L A N 65
13. (C)
The inference comes right out of the first sentence: If the only way a bookstore can
profitably sell books at below market prices is to get the books at a discount from
publishers, then it certainly must be true that a bookstore that is profitably selling books at
below market prices is getting discounts from publishers.
(A) and (B) both confuse necessity with sufficiency: For (A), its necessary to receive
discounts to sell at below-market prices, but its not a guarantee. As for (B), a high volume
relies on either exclusive access or a large specialized market (a necessary condition), but
neither of these things necessarily guarantees (i.e., is sufficient) a high sales volume.
(D) The bookstore in this choice (one that doesnt sell books at below-market prices) is
entirely outside of the scope of the argument, so nothing can be inferred about it.
(E) No; it's quite possible that a bookstore with exclusive access to a large specialized
market that also caters to mass tastes will be able to sell books at a discount.
The technical term for a very common LR mistake thats particularly related to formal
logic stimuli is denying the antecedent. You dont have to know this term, but you
should be familiar with the common error it signifies. Using classic algebraic form,
we know that we can take the statement If X, then Y and form the contrapositive by
reversing and negating the terms: If NOT Y, then NOT X. Denying the antecedent
involves negating the terms, but forgetting to reverse them, and the ensuing
statement If NOT X, then NOT Y, an example of which appears in choice (D) here,
is not logically valid. As stated above, NOT X, which in this case represents
bookstores that dont sell books at below-market prices, is outside the scope.
Weve seen it plenty on this test alone, and the concept comes up in different forms
all the time on the LSATmake sure you have a good grasp of the difference
between necessary and sufficient conditions.
14. (D)
Because this store doesnt cater to mass tastes, if it does not have exclusive access, then it is
impossible for that store to generate the volume to get the discounts that would allow the
store to profitably sell its books at below-market prices.
(A) could be true; all the store needs is exclusive access to a large specialized market.
(B) can literally always be true; nothing forces bookstores to sell at below-market prices,
after all.
(C) is certainly true. Since this store doesnt cater to mass tastes, then either it has exclusive
access to a large specialized market, or it can kiss the volume, and by extension, the
discounts, goodbye.
(E) Since this store doesnt cater to mass tastes, its certainly possible that the store doesnt
have exclusive access or the publishers discount.
LSAT PREP _______________________________________________________________ LSAT Test III Explained: Section IV
66 K A P L A N
While you should always read the question stem first, be careful with stimuli with
two questions. If the second stem introduces new information (like 14 does), be sure
that you dont take that information back with you to the first question. Just like in
Logic Games, new information is for that question only.
Draw upon your Logic Games skills for Logical Reasoning whenever possible. From
Games, you know how to handle a CANNOT be true questionsimply determine
whether each choice could or must be true, and cross those off.
15. (E)
Species have been coming and going long before humans ever came along, and those who
wish to blame recent extinctions on human technology and its consequent effects on the
environment ignore the fact that extinction is a natural process that would be going on
even if we were not around. In other words, the author argues that the more recent
extinctions are just part of the same process that has been going on since before the
environment was harmed by technology. However, she has failed to present any proof that
the more recently extinct species would have definitely become extinct without our help.
(A) is mired in a scope shiftthe author doesn't argue that technology has not harmed the
environment in any way; all she argues is that damage to the environment didn't cause
extinctions which wouldn't have otherwise occurred naturally.
(B) The author isn't ignoring this fact; it's just not important to her argument. These species
aren't mentioned because they have no effect on her argument, not because she erroneously
overlooks them.
(C) Once again, failing to consider something that has nothing to do with your argument is
not a reasoning error. The existence of undiscovered species has no bearing on the
argument that extinction is natural.
(D) is a distortion of the author's second statement. She doesn't identify a group of
scientists that have this theory, she just says that scientists, in general, estimate that it's the
case.
Pre-phrasing an answer can be particularly useful in identifying flaws. If you can
identify whats missing in the argument, actively search the choices for your idea.
More often than not, it will be there.
Once again, the author is not responsible for including in her argument every bit of
information that falls under the scope of her general topic. So dont be fooled by
choices that appear to be true, thinking Yeah, the author does fail to mention this.
The more appropriate test is yeah, the author doesnt mention this, but does she
have to? Does the argument depend on it? The faster you see that statements of fact
can be both true and the wrong answer to the question (in both LR and RC), the
better.
LSAT PREP _______________________________________________________________ LSAT Test III Explained: Section IV
K A P L A N 67
16. (A)
If the public distrusts the media, believing that it might be prejudiced, but they use
information dispersed by the media as fuel for that doubt, then it seems probable that the
public would find it difficult to detect a wide-spread media bias. In other words, it would
be difficult to find an objective standard against which to judge a media report.(A) is a
rephrasing of that thinking.
(B) The public may believe that the media is biased, and the media may indeed be biased,
but there's no evidence in the stimulus that there is a specific political agenda at work.
(C) The author hasnt even established that biases actually exist, just that the public may
have trouble discerning media bias, if it even exists. The reasons for the bias therefore
cannot be a conclusion the authors moving towards.
(D) Were only interested in the bias of the media and the fact that the public would have a
hard time seeing it. Whether or not reporters hold the same view as their public is
immaterial. Theres also a scope shift here: The views discussed in (D) dont necessarily
fall under the category of biases, which is after all what the stimulus talks about.
(E) We're given no clues as to how the public would respond to a scenario like this.
A conclusion will almost always cover the main topic and must certainly fall within
the scope of the argument. Here, the topic is biases in mass media, which only the
first three choices deal with. From there, paying attention to the scope of the
argument should have helped you to narrow it down to correct choice (A).
17. (D)
You should be able to pre-phrase a pretty close answer. If no risky projects are decided
upon in bureaucracies, and the bureaucratic decision making involves many people,
then the author is likely assuming that decisions to undertake risky projects are only made
by a single individual.
(A) Try the Denial Test: What if not all projects in a bureaucracy involve risk? Does the
argument suffer? No; the conclusion is about the fate of risky projects, but it doesnt rely
on the notion that all projects in a bureaucracy involve risksome may not without
affecting the logic of the argument.
(B) What type of people work in bureaucracies is beyond the scope, and certainly not a
crucial factor in making this argument.
(C) contains a few scope shifts: First,taking risks may be similar but is not necessarily the
same thing as undertaking risky projects. Second, the choice concerns an individual with
decision making power, whereas the stimulus indirectly refers to a situation where a single
individual has all of the power. If youre unsure, try the Denial Test again: Its possible for
an individual with decision-making power to not take risks, and for the conclusion to still
remain valid.
LSAT PREP _______________________________________________________________ LSAT Test III Explained: Section IV
68 K A P L A N
(E) says that people take fewer risks when they're members of a group. But we need a
statement that implies that they take no risks when they're part of a bureaucracy. So while
(E) may have initially appeared tempting, it's not strong or specific enough to allow the
argument's conclusion to follow from the premises.
The point of previewing the question stem is so that in a case like this, youll expect
to find a gaping hole in the argument.This advance notice should help you to pre-
phrase an answer before moving on to the choices.
18. (E)
Three things are required in order to achieve the physicalists' goal: 1) a knowledge of the
basic functions of neurons; 2) a knowledge of how neurons interact; and 3) a delineation of
the psychological faculties to be explained. At present, two of these things have already
been attainedwe have knowledge of the basic functions of neurons, and we understand
the scope and character of many psychological capacities. Therefore, the author concludes,
we can expect to achieve the goal of explaining mental functions in neurobiological terms
in the near future. What happened to a knowledge of how neurons interact? The flaw is
that three things are required to achieve a certain result, and the author concludes that the
result can be achieved even though we only have two.
(A) The conclusion agrees with, not contradicts, the physicalists.
(B) The author should not be expected to describe exactly what is currently known about
the basic functions of neurons. Were concerned only that that knowledge is needed for
the result, and that it currently exists. The arguments validity doesnt rely on a delineation
of the specific mechanics involved.
(C)'s complaint, that the word neurobiological is used as if it had the same meaning as
mental, is the very opposite of the truth. If the two words had been treated as meaning
the same thing from the beginning, the argument would have been pointless. The point is
that ultimately all mental functions will be explainable in neurobiological terms.
(D) Outside the scope; the argument isnt concerned with whether explaining mental
functions in neurobiological terms is useful, only that it will be accomplished in the near
future.
Transform the prose into simple thoughts and general terms whenever possible. All
thats going on here can be broken down into this: Something can be accomplished
if we have X, Y, and Z. We have X and Y, so it can be done. The flaw? What happened
to Z?
Remember the Can vs. Should scope shift: If the point of an argument is that
something can be done, whether its useful or it should be done is usually irrelevant.
Conversely, if an author argues that something should be done, demonstrating
whether its easy or difficult or even possible to do is usually not the authors
responsibility.
LSAT PREP _______________________________________________________________ LSAT Test III Explained: Section IV
K A P L A N 69
19. (D)
The author argues that the government can win back the lost administrators by raising
government salaries to a level that is comparable with private sector salaries. However, this
is possible only if these administrators will willingly change jobs again, and go back to the
government posts they left behind. If not, the argument falls apart.
(A) The individuals in question are already experienced, and the author feels that it is
already worth the extra money required for the government to recapture them, so any
new experience is not necessary to the worth of this plan.
(B) We learn in the first sentence that these administrators are both experienced and
extremely capable, and both of these qualities are important. Competence can be the most
important factor, as opposed to experience, in determining how well government agencies
function, and the argument would still be unaffected, which shows that this (B) is not a
necessary assumption on the part of the author.
(C) Try the Denial Test: The denial of (C) would be the government taking no action and
the gap in salaries not increasing. This state of affairs does nothing to damage the author's
proposal, which is about how to recapture administrators who have already left.
(E) is the opposite of what is assumed. The argument has said all along that the
administrators are chasing the big money. (E) would have them moving in large numbers
to the lower-paying (public) government jobs.
Many authors see the world as purely black and white, and often present arguments
in very simplistic terms that ignore alternative possibilities, implications,
interpretations, and solutions. Its no wonder that so many arguments on the LR
sections are flawed, can be weakened, need to be strengthened, or have gaping holes
in them (assumptions). Using your common sense and critical thinking skills to see
other sides of the story will win you many points.
20. (C)
The politician and his opponentsthose who want more funding for low-income
housingagree that homelessness is a serious social problem. They disagree about how to
solve it. Dont worry yourself with the obvious flaw in his argument. We arent asked that.
Focus on the phrase homelessness is a serious social problem; its simply information
that serves to set up the argument, but actually does little more. Understanding this allows
should help you eliminate choices that mistakenly ascribe special significance to this
statement, when in fact its little more than filler material. And since the argument itself is
based on other pieces of evidence, this intro statement is consistent with an acceptance or
denial of the conclusion.
(A) The statement is merely offered to frame the argument; its not an alternative
perspective to the one the politician adopts.
(B) The politician doesnt set out to solve the problem, he sets out to refute a proposed
solution.
LSAT PREP _______________________________________________________________ LSAT Test III Explained: Section IV
70 K A P L A N
(D) The politician admits that homelessness is a serious problem; he's certainly not trying
to discredit this position.
(E) It isn't necessary for the politician's conclusion. His argument that the problem of
homelessness can't be solved by providing more housing works just as well if
homelessness is a minor problem or no problem at all.
Again we see why reading the question stem first is vital and why we at Kaplan
continue to stress it. Knowing that we should concentrate on the phrase
homelessness is a serious social problem saves us from having to do another
reading of the stimulus.
Always read with the questions Why are they telling me this? and How crucial is
this information to the argument? in mind. In this case, recognizing that the phrase
in question just aint that important helps us to kill most of the wrong choices and to
see that its compatible with any judgment passed on the conclusion.
21. (B)
The reason that Thomas finds it incredible that such a small dietary change could have
such a drastic effect on population is because hes overlooked the fact that
overconsumption of eggs isn't the only cause of death. When Leona says that 5,000 lives
might be saved, she means that people who would otherwise have died from an
overconsumption of eggs would be spared. Who's to say that some of those people
wouldn't die for some other reason? Therefore, an adequate response from Leona would
focus on the fact that the numbers cited by her represent the number of people who
without the diet may have died, and with the diet wouldn't die, from eating too many
eggs; however, she never said that they wouldn't die for some other reason.
(A) and (D) discuss population growth (a major scope shift), which neither clarifies Leona's
claim nor addresses Thomas's point. Neither person is claiming the overall population
would grow by 5,000 people per year; Leona simply argues that 5,000 people who would
have died from eating too many eggs would not die for this reason.
(C) is irrelevant, as it addresses what would happen if egg consumption were cut by more
than half, not by half, which is the issue. Anyway, if Leona responded with choice (C), she
would only confuse poor Thomas even more, because then his 50,000 figure, which he
thought was too high to begin with, would have to be even higher.
(E) makes no sense as a response to Thomas. The issue isn't what individual consumers
must do to comply with this dietary changeits what will be the result of the dietary
change.
In a dialogue question, keep in mind the person with which the question is
concerned. Arguments that Thomas could make are of no interest to us here.
As always, decode the stem when its more complex than usual, and if youre asked
to do two things, make sure the answer you choose accomplishes both.
LSAT PREP _______________________________________________________________ LSAT Test III Explained: Section IV
K A P L A N 71
22. (B)
The key notion in this one is that only after that transfer occurs can the new therapies help
patients. The inference turns out to be pretty straightforward. If the new therapies can help
patients only after they have been transferred from the lab to the marketplace, then they cant
possibly help patients before that.
(A) We don't know that the FDA necessarily regulates all therapeutic agents after they've
been put on the market. We're only told that the FDA regulates the actual introduction of these
agents.
(C) All we're told is that the research community carries out a long process of discovery and
testing. For all we know, though, it's the FDA that is responsible for thisperhaps the agency
requires the long testing period.
(D) Maintaining the quality of therapeutic agents is a new subject altogether, as is the issue of
what the FDA should, or shouldn't, do.
(E) Heres the necessary/sufficient thing again: Only after a new drug has been introduced
can it help patients. That is not to say that it necessarily will help patients.
A word like only can alter the entire meaning of a stimulus. When you see this word,
it tells you that a necessary condition is being introduced. As evidenced here, answers
can spring entirely from an only statement.
23. (A)
The question stem introduces a conclusion that could lead from this stimulus; that
automobile theft has been reduced by the program. Were asked for the choice that asks the
question whose answer is most important in evaluating that conclusionthat is, whose
answer is most likely to tell us whether or not the conclusion is justified. Since the conclusion
in the stem is based on the fact that cars bearing the special decals have a lower theft rate than
other cars, we can expect that the correct question will ask if there might not be another reason
that these cars aren't being stolen, besides the allegedly successful program (theres that
alternative explanation concept popping up again). (A) does this by asking if the car
owners that are likely to join the program owners take any other special precautions to
prevent theft. If the answer to this question is yes, then perhaps the other measures, not the
program, are keeping the number of thefts down, which would signify that the stems
conclusion that the program has reduced thefts is unwarranted.
(B), (E) How many neighborhoods the program is operating in (A) and whether
neighborhoods in which the program took effect were a representative cross-section of all
neighborhoods, in terms of car types (E), are both irrelevant. The conclusion only claims that
the program was successful in the neighborhoods where it was actually applied.
(C) The program only takes effect between 1 A.M. and 5 A.M., so what happens during the
daytime can have nothing to do with the program. Moreover, the issue here is the cars that
weren't stolen, and the reasons why they weren't stolen.
LSAT PREP _______________________________________________________________ LSAT Test III Explained: Section IV
72 K A P L A N
(D) While this certainly might be of some concern to the owners, it doesnt say much about
the effectiveness of the program. True, if the answer is yes, it would indicate that the program
is indeed functioning, but doesnt help us to evaluate its effectiveness as well as (A) does, the
answer of whih could blow the conclusion out of the water.
Once again, you need to translate, or transform the stem into a workable framework.
This is basically an offshoot of a strengthen/weaken question; you need to determine
whether the answer to the question posed in each choice will significantly damage or
shore up the argument. It doesnt matter which way it goes, since an evaluation could
go either way. If the answer to a question greatly strengthens or weakens the argument,
then that question is useful in evaluating the argument.
24. (E)
Break the principle down to its essentials: An action is morally good only if it 1) benefits
another person and 2) was intended to benefit that other person. An action is morally bad if it
harms another person, and either 1) such harm was intended or 2) a reasonable person should
have known that harm was likely to occur. The only way to find the situation that matches the
spirit of this principle is to work through the choices, and unfortunately, the testmakers
buried the answer in choice (E). Jonathan's act of neglecting his three-year-old niece caused
harm to her, so it meets the first criterion for a morally bad act. Although he intended no
harm, he should have realized that his failure to watch his niece carefully was likely to lead
to harm; (E)'s judgment therefore conforms to the principle in the stimulus.
(A) According to the stimulus, an action is only morally bad if it actually causes harm
which the action in this case did not.
(B) Jeffrey's action indirectlyand thats the key wordhelped Sara. Since his action was
performed with the intention of securing his own promotion, not with the intention of
benefiting Sara, it fails the stimulus' test for a morally good action.
(C) Once again, we have an action that fails the morally good test on the grounds that it
doesnt benefit anyone (quite the opposite, no less).
(D) The homeless man was obviously harmed, which puts Marilees action in the morally
bad ballpark. But it still needs to pass another testthe harm must be inflicted intentionally,
or the harmful result must be foreseeable. Neither of these are met here (what are the chances
that someone will choke on a sandwich; it was most likely the mans own fault for talking and
chewing at the same time), so the judgment that Marilees performed a morally bad action
doesnt conform to the principle in the stimulus.
Take control of the test. This question has possibly time consuming written all
over it. It just happens to come at the end of the section, but if it didnt it may have
been a very good candidate to skip temporarily and come back to it time permitted.
I.N. LL3107 Rev.A Printed in the USA

You might also like